Flaps 01-22 Flashcards

1
Q

An 18-year-old man is scheduled to undergo free fibula flap for mandible reconstruction. The patient reports that he is a track and field athlete. Regarding the function of his great toe, which of the following is the most likely outcome?

A) Abduction deformity
B) Decreased flexion strength
C) Neuropathic pain
D) Normal extension

A

The correct response is Option B.

The fibula free flap is commonly used for mandibular reconstruction because of the availability of up to 25 cm of bone. However, donor site morbidity is a significant consideration and is common with this flap. Common motor morbidity includes impaired flexor hallucis longus (FHL) function, with a significant reduction in strength between the affected and unaffected legs. As a result of loss of the FHL and decreased opposition to the long toe flexors, claw toe deformity and weakened dorsiflexion are also common. Gait may be affected by these deformities.

Sensory deficits typically include the peroneal nerve distribution and may cause pain at the operative site or sensory deficits over the dorsum of the foot.

How well did you know this?
1
Not at all
2
3
4
5
Perfectly
2
Q

A 56-year-old man presents to the emergency department with fever and neck pain 2.5 weeks after undergoing posterior cervical spine fusion. Clinical examination shows peri-incisional edema and erythema. CT scan shows a deep posterior fluid collection. The orthopedic surgery team plans surgical treatment with debridement and washout and consults the plastic surgeon for obliteration of an anticipated dead space. A Mathes and Nahai type II muscle flap is planned for reconstruction. Which of the following arteries is most likely involved in this reconstruction?

A) Circumflex scapular
B) Occipital
C) Thoracoacromial
D) Thoracodorsal
E) Transverse cervical

A

The correct response is Option E.

The transverse cervical artery is the main pedicle to the trapezius muscle flap (type II muscle), which is particularly suitable for high cervical defects since the paraspinous muscles are of limited size and mobility at this level. The main pedicle of the trapezius flap enters the deep aspect of the muscle approximately 8 cm from the midline at the level of C7. Following identification of the main pedicle, the lateral aspect of the muscle is divided to permit transposition into the defect.

Postoperative wound complications following spinal procedures approach 40% in high-risk patients. Thus, combined reconstructive approaches with orthopedic surgery and neurosurgery are common for management of these complex soft-tissue defects. Key management principles include timely debridement, bony fixation, and dead space obliteration. Therefore, muscle flaps are particularly suitable for reconstruction.

A branch of the occipital artery is the dominant pedicle to the sternocleidomastoid flap, which is not a suitable reconstructive option for coverage of a posterior cervical defect. The circumflex scapular artery is the dominant pedicle to the scapular/parascapular fasciocutaneous (not muscle) flaps. The thoracoacromial artery is the dominant pedicle to the pectoralis major muscle (type V muscle). The thoracodorsal artery is the dominant pedicle to the latissimus dorsi muscle (type V muscle).

How well did you know this?
1
Not at all
2
3
4
5
Perfectly
3
Q

A 24-year-old man presents with a burn contracture of the third web space of the hand. A jumping man (five flap) plasty is planned. In this technique, the final position of the central flap is achieved through which of the following techniques?

A) 45-degree transposition
B) 60-degree transposition
C) 75-degree transposition
D) V-Y advancement
E) Y-V advancement

A

The correct response is Option E.

In the jumping man or five flap plasty, the central flap is advanced through a Y to V method where the tip of the V is advanced into an incision made in the opposing tissue. There are different modifications to this technique for local flap lengthening of flexion contractures. Traditionally, the lateral limbs are designed with 60-degree angles to the central limb and the opposing limbs are designed more obtusely to prevent ischemia at the base. This type of reconstruction requires laxity of the surrounding skin and is therefore most often employed in places like the medial canthus, web spaces of the feet and hands, or the axilla.

A V-Y advancement is commonly used in pressure ulcer repair where the base of the V is advanced into the defect to create a Y. This is the opposite of what one is trying to achieve in a jumping man plasty. Transpositions of 75, 60, and 45 degrees are all different variations of z-plasties and are utilized in this local rearrangement, but the central flap is advanced, not transposed/rotated.

How well did you know this?
1
Not at all
2
3
4
5
Perfectly
4
Q

An otherwise healthy 55-year-old man is diagnosed with a soft-tissue sarcoma of his mid-anterior leg. Neoadjuvant radiation therapy is performed, followed by wide local excision. A superficial circumflex iliac perforator-based flap is planned for reconstruction of the 10 × 5-cm defect. Which of the following is the main advantage of this flap over a radial forearm flap?

A) Ability to neurotize the flap
B) Ease of flap harvest
C) Length of pedicle
D) Lower donor site morbidity
E) Thickness of flap

A

The correct response is Option D.

The main advantage of the superficial circumflex iliac perforator-based (SCIP) flap over the radial forearm flap (RFF) is the improved donor site outcome. The SCIP flap can be primarily closed in most cases, whereas most RFF donor sites need a split skin graft to get closure. Additionally, the groin donor site is in a relatively hidden area compared with the RFF. The SCIP flap is a more difficult flap to raise compared with the RFF, and also has a smaller diameter and length of pedicle. Both flaps are known for being raised as thin flaps and can potentially be neurotized, with the SCIP utilizing an intercostal nerve branch, whereas the RFF uses the lateral and/or medial antebrachial cutaneous nerve(s).

How well did you know this?
1
Not at all
2
3
4
5
Perfectly
5
Q

A 65-year-old woman presents with a malignant melanoma of the right ankle that requires resection resulting in exposed bone. Her thighs are too bulky as donor sites for ankle skin resurfacing. Reconstruction with a free fascio-cutaneous scapular flap is planned. Through which of the following anatomic landmarks will the pedicle for this flap pass?

A) Anterior triangle of the neck
B) Posterior triangle of the neck
C) Quadrangular space
D) Subclavian triangle
E) Triangular space

A

The correct response is Option E.

The pedicle for the scapular fasciocutaneous flap is the circumflex scapular artery arising from the subscapular system. The circumflex scapular artery travels through the triangular space bordered laterally by the long head of the triceps, the teres minor above and teres major below. The posterior circumflex humeral vessels travel within the quadrangular space, which is adjacent to the triangular space and is bordered by the teres minor above, teres major below, long head of the triceps medially, and the humerus laterally. The anterior triangle of the neck is bordered superiorly by the mandible, laterally by the sternocleidomastoid muscle, and medially by the midline of the neck. The contents of the anterior triangle include cranial nerves VII, IX to XII, and the carotid arteries and internal jugular vein. The posterior triangle of the neck is bordered by the trapezius muscle posteriorly, the sternocleidomastoid muscle anteriorly, and the middle third of the clavicle inferiorly. The posterior triangle of the neck contains the external jugular vein, the brachial plexus, regional lymph nodes, third part of the subclavian artery, as well as the suprascapular vessels and cervical branches of thyrocervical trunk. The posterior triangle of the neck is divided by the inferior belly of the omohyoid muscle into the occipital triangle above and subclavian triangle below. The subclavian triangle is bounded by the sternocleidomastoid, omohyoid, and the clavicle.

How well did you know this?
1
Not at all
2
3
4
5
Perfectly
6
Q

A 28-year-old woman presents to the clinic 3 years after she sustained large, full-thickness burns to the anterior neck. She has undergone reconstruction with a fasciocutaneous free flap. She does not like the contracture and webbing along the margin of the reconstruction. Two contiguous 45-degree Z-plasties are planned over a total length of 10 cm. The expected increase in scar length is which of the following?

A) 2.5 cm
B) 5.0 cm
C) 7.5 cm
D) 10.0 cm
E) 12.5 cm

A

The correct response is Option B.

A 45-degree Z-plasty is expected to lengthen the total scar by 50%. If each Z-plasty covers 5 cm of scar, then each Z-plasty will increase scar length by 2.5 cm, for a total increase of 5 cm. Theoretical gains in length for Z-plasty angles are as follows:

The Z-plasty was the first flap introduced using a mathematical approach to correct a skin defect or scar. A standard Z-plasty design uses three incisions of equal length and two angles of equal degree. The primary principle is to transfer lateral skin excess to lengthen and reorient tight scars or contractures. The central incision is oriented parallel to the long axis of the scar, or the scar may be incorporated as an excision acting as the central incision. The resultant triangular skin flaps are transposed with each other, resulting in a new central incision that is perpendicular to the original orientation.

How well did you know this?
1
Not at all
2
3
4
5
Perfectly
7
Q

A 32-year-old man sustains a burn to his hand, resulting in a first web space contracture. A four-flap Z-plasty is planned for release of the contracture. Which of the following best describes the flap reconstruction for this defect?

A

The correct response is Option F.

Random pattern flaps are generally classified based on the primary motion of the flap. There are three basic types of tissue movement: transposition, advancement, and rotation. Transposition flaps incorporate noncontiguous skin into a defect by lifting the flap over normal skin for inset into a defect. Advancement flaps recruit adjacent tissue to close a defect via soft-tissue movement in a linear direction. Rotation flaps move adjacent tissue around an axis to close a defect, rotating soft tissue into the defect. Advancement and rotation flaps recruit adjacent lax tissue and move in either a linear or arced motion (respectively) to fill the defect. Transposition flaps recruit noncontiguous tissue, which is lifted over intact soft tissue and placed into the defect. Random pattern flaps lack a defined named arterial vascular supply. Because of their lack of an axial vascular supply, they are subject to dimensional restrictions. In general, they are designed to not exceed a length:width ratio of 2:1. An axial pattern flap is a single pedicled flap that has an anatomically named arterial blood supply running along its long axis. Because of the presence of a named arterial blood supply, axial pattern flaps are not subject to the length:width ratio restrictions that apply to random pattern flaps.

How well did you know this?
1
Not at all
2
3
4
5
Perfectly
8
Q

A 65-year-old woman presents with a 4 x 4-cm elliptical defect on the vertex of the scalp after resection of a trichilemmal cyst. Local advancement flaps are planned for closure with wide undermining. Division of which of the following layers is important during scalp flap advancement?

A) Dermis
B) Galeal aponeurosis
C) Innominate fascia
D) Pericranium
E) Subgaleal fascia

A

The correct response is Option B.

During a scalp advancement flap, release of the galeal aponeurosis is important, as this layer provides the majority of resistance to scalp advancement. Often scoring of the galea is performed in parallel 1-cm intervals to provide adequate release.

The layers of the scalp include: skin, dense connective tissue, galeal/epicranial aponeurosis, loose areolar connective tissue, and pericranium. Release of the other layers listed is not effective, as they do not provide appreciable resistance. The innominate and the subgaleal fascia are the same layer, otherwise known as the subaponeurotic layer or the loose areolar layer. Release of this layer does not offer appreciable laxity when performing advancement flaps.

How well did you know this?
1
Not at all
2
3
4
5
Perfectly
9
Q

A 5-year-old boy presents to the emergency department 4 hours after he sustained an amputation of his left index finger when it was slammed in a door. The parents brought the amputated digit in a plastic bag on ice. The amputation is at the level of the mid proximal phalanx. Which of the following is the most important reason to attempt replantation?

A) The amputation is proximal to the flexor digitorum superficialis insertion
B) The cold ischemia time is less than 6 hours
C) It is the index finger
D) It is a single-digit amputation
E) The patient is a child

A

The correct response is Option E.

Digital replantation should almost always be attempted in a child, except when the amputated part is severely crushed or there are other life-threatening injuries that preclude surgery. Replantation in children is technically more challenging due to the smaller size of the vessels. However, functional outcomes are more superior than in adults. The replanted parts have better sensory return and can have normal growth. Amputations through joints also exhibit remarkable joint remodeling.

A single digit amputation, especially proximal to the flexor digitorum superficialis (FDS) insertion is considered a contraindication to replantation. Digit replantations proximal to the FDS insertion have a poor range of motion as compared to amputations distal to the FDS insertion. This is, thus, an important landmark when making decisions about amputation versus replantation. Multiple digit amputations are an indication for replantation as the functioning deficit with loss of multiple digits is great. The thumb is responsible for 40% of the function of the hand and should always be replanted, if possible. Even if it is stiff and insensate, a replanted thumb will act as a post for opposition.

Index finger amputations at or proximal to the proximal interphalangeal joint are considered by many to be an indication for amputation. A stiff and painful index finger is likely to be excluded by the patient; amputation will result in better global hand function.

Digits tolerate longer ischemia times than more proximal level amputations, due to absence of muscle. Amputated digits tolerate warm ischemia times of 6 to 12 hours and cold ischemia times of 12 to 24 hours. Digital replantation has been reported with warm ischemia time of 33 hours and cold ischemia time of 94 hours. Cold ischemia time is thus not a major consideration in the decision-making process for amputation versus replantation.

How well did you know this?
1
Not at all
2
3
4
5
Perfectly
10
Q

To reconstruct a traumatic soft-tissue defect of the hand, a lateral arm fasciocutaneous flap is chosen for free tissue transfer. The vascular pedicle for this flap is located between which of the following structures?

A) Brachioradialis and brachialis
B) Lateral head of the triceps and biceps
C) Lateral head of the triceps and the brachialis
D) Lateral head of the triceps and the extensor carpi radialis longus
E) Lateral head of the triceps and the humerus

A

The correct response is Option C.

The lateral arm flap is a reliable fasciocutaneous flap used for reconstruction of small to medium size soft-tissue defects of the forearm and hand. It provides thin tissue of excellent quality and a satisfactory aesthetic outcome with the ability for primary closure of the donor site.

The dominant pedicle is the posterior radial collateral artery. This vessel originates from the radial collateral artery which is a branch of the brachial artery in the upper arm and emerges between the brachialis muscle anteriorly and the lateral head of the triceps posteriorly to supply the skin and soft tissue of the lateral arm.

In addition, the anatomic relationships between the vascular pedicle and radial nerve are important to understand when dissecting this flap. The radial nerve courses posterior to the humerus before wrapping around the lateral aspect of the humerus from posterior to anterior. The antebrachial cutaneous nerve of the forearm branches from the radial nerve to join the vascular pedicle (posterior branch of radial collateral artery). This cutaneous nerve can be preserved with careful dissection and helps to identify the radial nerve proximally during flap elevation. Segmental perforating vessels emerge within the septum separating the triceps and brachialis muscles.

How well did you know this?
1
Not at all
2
3
4
5
Perfectly
11
Q

A 44-year-old man presents with a 20-degree extension deformity of his wrist because of burn scarring. Z-plasty for contracture release is planned. Which of the following (A-D) is the most appropriate placement of the central limb for this procedure in this patient?

A) (see image above)
B) (see image above)
C) (see image above)
D) (see image above)

A

The correct response is Option A.

Z-plasty is a fundamental and common reconstructive technique used to elongate scars or contractures, narrow scars, rearrange tissues into relaxed skin tension lines, camouflage scars, or releasing tension. This is a form of rotation and advancement whereby a central limb of the Z-plasty is drawn parallel to the line of maximal tension, and subsequent limbs are drawn anywhere from 30 to 90 degrees from this. Wider angles give greater scar elongation at the expense of greater transverse tension.

In this scenario, the line of maximum tension is longitudinal, creating an extension deformity. Maximal contracture release will occur with a central limb of the Z-plasty drawn longitudinally.

How well did you know this?
1
Not at all
2
3
4
5
Perfectly
12
Q

A 43-year-old man presents for reconstruction of a soft tissue deficit of the antecubital fossa with a reverse lateral arm pedicled flap. Which of the following arteries is the blood supply for this flap?

A) Anterior interosseous
B) Persistent median
C) Posterior interosseous
D) Radial recurrent
E) Ulnar

A

The correct response is Option D.

Although the lateral arm flap has predominantly been used in free tissue transfer for distant defects based on the posterior radial collateral artery, transfer as a pedicled reverse-flow flap based on the radial recurrent artery has been both anatomically and clinically proven. Occasionally, it is performed with a delay procedure at an intermediate stage.

The anterior and posterior interosseous arteries can provide circulation to perforator flaps. The ulnar artery has been occasionally used for an ulnar artery based fasciocutaneous flap. The persistent median artery passes through the carpal tunnel and runs with the median nerve.

How well did you know this?
1
Not at all
2
3
4
5
Perfectly
13
Q

A 10-year-old boy with osteosarcoma is undergoing resection of 8 cm of proximal tibia. Reconstruction with a vascularized bone flap is planned. Which of the following arteries is the primary blood supply for this flap?

A) Anterior tibial
B) Dorsalis pedis
C) Peroneal
D) Popliteal
E) Posterior tibial

A

The correct response is Option C.

The peroneal artery is adjacent to the fibula. It arises from the tibioperoneal trunk, immediately distal to the takeoff of the anterior tibial artery. It perforates the interosseous membrane. The peroneal artery also gives perforators to the skin of the lower leg. The length of the pedicle is usually short, but can be increased substantially by dissecting the peroneal artery from the fibula and using the distal bone for reconstruction.

The popiteal artery is proximal to the other arteries mentioned. After crossing the knee, it branches into the anterior and posterior tibial arteries. The posterior tibial artery then gives off the peroneal artery.

How well did you know this?
1
Not at all
2
3
4
5
Perfectly
14
Q

A 75-year-old man with a history of renal transplantation presents with a rapidly growing squamous cell carcinoma involving his anterior scalp and forehead. A photograph is shown. Wide excision results in a 20 × 20-cm defect with exposed calvarium. Which of the following flaps is most appropriate for coverage of this wound?

A) Anterolateral thigh free flap
B) Gracilis free flap
C) Lateral arm free flap
D) Radial forearm free flap
E) Rectus abdominis free flap

A

The correct response is Option A.

The anterolateral thigh (ALT) free flap (shown) is the only option listed with the requisite tissue characteristics, size, and pedicle length necessary to repair a defect of this size and location. The ALT flap has rapidly become a first-line option for covering large or irregular head and neck defects. It is an extremely versatile flap with chimeric options allowing for multiple skin paddles, muscle bulk (vastus lateralis) if needed for dead space obliteration, as well as bone (iliac crest) for midface or small mandibular defects. The tissue thickness is well suited for large scalp defects. Flaps as large as 35 × 25 cm have been described, although defects larger than 10 to 12 cm in width generally require skin grafting the donor site. Pedicle lengths of 12 to 16 cm have been described, which is often long enough to reach the upper neck if necessary.

The radial forearm free flap is an excellent option in many cases, providing a pedicle length up to 18 to 20 cm with maximal dimensions up to 12 × 30 cm. A major criticism of this flap is the poor cosmesis of the donor site and lack of subcutaneous tissue necessary to match the surrounding cutaneous defect. The lateral arm flap based on the radial collateral artery is useful for smaller defects in the head and neck but is limited in this situation by a short pedicle (6 cm) and smaller skin paddle (6 × 12 cm).

Muscle flaps covered with skin grafts have also been described for scalp coverage. When necessary, a latissimus dorsi flap is a great option for near total scalp coverage. With the examples listed here, the gracilis flap is too narrow (5 to 6 cm) with too short a pedicle (medial femoral circumflex, 7 cm). While the pedicle for the rectus free flap (deep inferior epigastric, 8 to 10 cm) might be long enough in this situation, the muscle is not big enough (6 cm wide) to cover the defect.

How well did you know this?
1
Not at all
2
3
4
5
Perfectly
15
Q

A 4-year-old girl is evaluated for an axillary skin contracture from a burn. Z-plasty is planned to lengthen the scar in order to improve the contracture. If 45-degree angles are planned, the most likely expected increase in the length of the scar would be which of the following?

A) 25%
B) 50%
C) 75%
D) 100%
E) 120%

A

The correct response is Option B.

The most likely expected increase in scar length is 50%. A Z-plasty is a technique using two triangular flaps which are interdigitated, producing a gain in length of the central limb placed along the line of contracture. The angles of the Z-plasty range from 30 to 90 degrees. The wider the angles, the greater the lengthening. A 30-degree angle is equal to a 25% increase in length, a 45-degree angle is equal to a 50% increase in length, a 75-degree angle is equal to a 100% increase in length, and a 90-degree angle is equal to a 120% increase in length. The most commonly used angle is 60 degrees, giving a 75% increase in length. This angle is optimal because angles less than 60 degrees may not provide enough lengthening, and angles greater than 60 degrees can produce significant tension inhibiting flap transposition.

How well did you know this?
1
Not at all
2
3
4
5
Perfectly
16
Q

A 35-year-old man is brought to the emergency department with a 15-cm open wound on the left hip and thigh after he was involved in an accident while using machinery at a construction site. Much of the skin of the lateral thigh is injured, and exposed bone is noted over the trochanter of the hip. Wound coverage using an anterolateral thigh flap from the right side is planned. When the flap is harvested, which of the following muscles must be identified in order to preserve perforators to the flap?

A) Gracilis
B) Inferior gluteal
C) Sartorius
D) Tensor fascia lata
E) Vastus lateralis

A

The correct response is Option E.

The anterolateral thigh flap is a versatile coverage tool because of its wide skin island (up to 8 x 25 cm) and long, accessible pedicle (up to 7 cm). The blood supply originates from the lateral femoral circumflex artery descending branch, and sends perforating branches through the vastus lateralis and rectus femoris muscles, and occasionally through the intermuscular septum.

The inferior gluteal muscle, while a common muscle flap, is further posterior and proximal. The tensor fascia lata is more lateral to the anterolateral thigh flap zone, though it also has a blood supply from the lateral femoral circumflex system, as the vessel terminates in the tensor fascia lata. The sartorius is more medial and proximal and has a segmental circulation based on the femoral artery branches. The gracilis is more medially based, and is supplied by the medial femoral circumflex.

How well did you know this?
1
Not at all
2
3
4
5
Perfectly
17
Q

A 40-year-old man sustains burns to 35% of his total body surface area, including the neck, chest, axillae, and upper extremities. After subsequent skin grafting, a right anterior axillary dome scar contracture develops. The patient is scheduled to undergo revision of the scar using Z-plasty. Which of the following lateral limb angles will result in a theoretical 75% gain in central limb length?

A) 30 degree
B) 45 degree
C) 60 degree
D) 75 degree
E) 90 degree

A

A 40-year-old man sustains burns to 35% of his total body surface area, including the neck, chest, axillae, and upper extremities. After subsequent skin grafting, a right anterior axillary dome scar contracture develops. The patient is scheduled to undergo revision of the scar using Z-plasty. Which of the following lateral limb angles will result in a theoretical 75% gain in central limb length?

A) 30 degree
B) 45 degree
C) 60 degree
D) 75 degree
E) 90 degree

The correct response is Option C.

The traditional standard Z-plasty consists of at least three incisions of equal length (two limbs and one central incision) and two angles of equal degree. Ideally, the central incision runs parallel to the long axis of the scar, or the scar itself may be completely excised with the fusiform defect acting as the central incision.

The resultant subcutaneous triangular skin flaps are transposed with each other such that the new, central incision lies perpendicular to the original central incision. After closure, the scar is reoriented along the limb incisions, and the new central incision lies within relaxed skin tension lines. The length of the original scar also increases after a Z-plasty, which is a useful characteristic when a surgeon desires release of a scar contracture, as in this specific example. In general, as the central incision lengthens (given a constant angle), so does the resultant scar. Additionally, as the angles between the limbs increase (given a constant limb length), so does the resultant scar.

How well did you know this?
1
Not at all
2
3
4
5
Perfectly
18
Q

A 63-year-old man has a full-thickness scalp defect following resection of a melanoma. Final pathology has confirmed clear margins. Which of the following criteria is an indication for coverage of the defect with a flap instead of a skin graft?

A) Alopecia of the surrounding skin
B) Exposed calvarium
C) Granulation tissue in the base of the wound
D) Intact pericranium
E) Posterior location

A

The correct response is Option B.

Exposed bone does not provide an adequately vascularized bed for skin graft take. Pericranium, in contrast, can support a skin graft. The presence of granulation tissue is a good sign that the wound bed is adequately vascularized for a skin graft to take. Surrounding alopecia decreases aesthetic concerns associated with reconstructive options that do not support hair growth. Defect location does not significantly affect the need for vascularized coverage.

How well did you know this?
1
Not at all
2
3
4
5
Perfectly
19
Q

Which of the following vessels runs within the pedicle of the reverse sural artery flap and serves as primary venous drainage?

A) Femoral
B) Greater saphenous
C) Lesser saphenous
D) Popliteal
E) Posterior tibial

A

The correct response is Option C.

The reverse superficial sural artery flap (RSSAF) is a distally based fasciocutaneous or adipofascial flap that is increasingly being used for coverage of defects that involve the distal third of the leg, ankle, and foot. First described by Donski and Fogdestam and later championed by Masquelet et al, RSSAF has become a popular option for these difficult wounds. The description of the RSSAF (Masquelet flap) has revolutionized the osteoplastic armamentarium of surgeons not conversant with microvascular free flaps. The reliability of septocutaneous perforators has been well documented. Hence, raising a flap based on this reliable anastomosis of peroneal artery and median sural artery, along with the sural nerve and lesser (short) saphenous vein has been described to be successful.

A significant advantage of this flap is that it does not require sacrifice of a major artery to the lower limb. Touted for its ease of dissection, the RSSAF is often reputed to have a favorable complication profile as evidenced by a recent meta-analysis that found 82% of flaps heal without any flap-related complications. The main complications include venous congestion of the flap requiring delay or leech therapy in some higher-risk patients.

The greater saphenous vein runs proximal and medial to the lesser saphenous vein, and it drains the medial and anteromedial portion of the lower leg.

The popliteal vein drains the lesser (short) saphenous vein, and it is therefore not the primary drainage of the reverse sural flap. The anterior and posterior tibial veins are the deep venous drainage of the lower leg and do not drain the RSSAF.

The femoral vein is the deep venous drainage system in the upper leg.

How well did you know this?
1
Not at all
2
3
4
5
Perfectly
20
Q

A 75-year-old man who had femoral-popliteal bypass surgery 2 weeks ago has an infection in the proximal groin. A muscle flap to fill the dead space is planned. Which of the following muscle flap options has a type IV Mathes-Nahai (multiple segmental vascular pedicles) vascular anatomy?

A) Gracilis
B) Rectus abdominis
C) Rectus femoris
D) Sartorius
E) Vastus medialis

A

The correct response is Option D.

The sartorius muscle classically has a type IV Mathes-Nahai vascular anatomy, which may limit its arc of rotation. In a recent study, even though the sartorius muscle has multiple segmental pedicles, there tend to be codominant superior and inferior pedicles that could possibly allow for the majority of the muscle to be raised on either the superior or inferior dominant pedicle.

Mathes-Nahai vascular anatomy classification for muscle flaps:

I – Single dominant vascular pedicle

II – Single dominant vascular pedicle with secondary minor vascular pedicles

III – Codominant major vascular pedicles

IV – Multiple segmental vascular pedicles

V – Dominant vascular pedicle with segmental secondary pedicles that can supply muscle if dominant is divided

Type II vascular anatomy is seen with the rectus femoris, vastus medialis, and gracilis muscles. The rectus abdominis has a type III vascular anatomy. Type V would be a latissimus dorsi muscle flap.

How well did you know this?
1
Not at all
2
3
4
5
Perfectly
21
Q

A 76-year-old woman with a history of left modified radical mastectomy and radiation therapy comes to the office because of a chronic wound of the left axilla associated with limitation of abduction and exposed rib at the wound base. A photograph is shown. Examination of a biopsy specimen excludes malignancy. In addition to appropriate debridement, which of the following is likely to be most effective in achieving wound closure?

A) Adjacent tissue transfer
B) Left latissimus dorsi myocutaneous flap
C) Negative pressure wound therapy
D) Radial forearm fasciocutaneous free flap
E) Split-thickness skin graft

A

The correct response is Option B.

The best option to achieve wound closure in this patient is an ipsilateral latissimus dorsi myocutaneous flap. For chronic wounds in an irradiated field, the best option is debridement followed by transfer of healthy, nonirradiated tissue. Negative pressure wound therapy is likely to result in a recurrent chronic wound, albeit a clean one. Split-thickness skin graft would be inappropriate in an irradiated wound bed with exposed bone. Autologous fat grafting can help improve the quality of irradiated tissues in the absence of a wound, and some studies have shown promise in the treatment of superficial radiation ulcers; however, this patient has necrotic rib, and following debridement the wound will be deep and large. Although wound management and fat grafting have been shown to promote healing in isolated cases, this approach is not yet an accepted standard of care. Adjacent tissue transfer will employ irradiated tissue, and is thus prone to necrosis, wound breakdown, and recurrent chronic wound formation. A free flap could be an option, but a forearm flap would not have the volume required for the expected defect. In addition, a free flap is more morbid than a local pedicled flap in this elderly patient.

How well did you know this?
1
Not at all
2
3
4
5
Perfectly
22
Q

A 55-year-old man who recently underwent a cardiac bypass procedure has a sternal infection that requires debridement. The defect is evaluated, and reconstruction using an omental flap is planned. Which of the following vessels provides the blood supply for this flap?

A) Gastroepiploic
B) Left gastric
C) Right gastric
D) Short gastric
E) Superior mesenteric

A

The correct response is Option A.

The omental flap is supplied by the gastroepiploic vessels. Common options for sternal wound reconstruction include the pectoralis major, rectus abdominis, latissimus dorsi, and omental flaps. The use of an omental flap for a mediastinal defect was described in the 1970s; however, muscle flaps became a popular choice for reconstruction in the 1980s. Based on the size of the defect, the omental flap can be used with or without a skin graft. The omentum has angiogenic and immunogenic properties that make it ideal for reconstruction of sternal wound infections.

The omentum is based on the left and right gastroepiploic vessels. In order to increase length, the flap can be based on one set of vessels, usually the right gastroepiploic vessels. The left gastroepiploic vessels are a branch of the splenic vessels; the right gastroepiploic vessels are a branch of the gastroduodenal vessels. Harvest can be performed through either an upper abdominal incision, transdiaphragmatic, or laparoscopically. There is a risk of donor site morbidity such as abdominal wound infections or symptomatic hernias.

The superior mesenteric vessels supply the lower part of the duodenum extending to the middle third of the transverse colon, as well as the pancreas. The left and right gastric vessels supply the lesser curvature of the stomach. The short gastric vessels supply a portion of the greater curvature of the stomach and are branches of the splenic vessels. The left and right gastroepiploic vessels supply the greater curvature of the stomach along with the omentum.

How well did you know this?
1
Not at all
2
3
4
5
Perfectly
23
Q

A 23-year-old man presents 2 years after sustaining full-thickness burns on the anterior neck. He has undergone tissue expansion and local flap reconstruction of the burn defect. He notes webbing and contracture at the margin of one of the prior flap reconstructions. Three identical 60-degree Z-plasties are planned over a total length of 12 cm. The expected gain in scar length is which of the following?

A) 3 cm
B) 4 cm
C) 6 cm
D) 8 cm
E) 9 cm

A

The correct response is Option E.

A 60-degree z-plasty lengthens a scar by 75%. If each z-plasty covers 4 cm of scar, each will lengthen the scar by 3 cm, for a total increase of 9 cm. In contrast, a 30-degree z-plasty lengthens an incision by 25%, and a 45-degree z-plasty lengthens an incision by 50%. To prevent undue tension, angles greater than 60 degrees should be avoided.

How well did you know this?
1
Not at all
2
3
4
5
Perfectly
24
Q

A 24-year-old man comes to the emergency department because of a dorsal hand injury. Physical examination shows a 6 × 4-cm full-thickness defect with exposed metacarpal bones. A medial sural artery perforator flap for soft-tissue coverage is planned. From which of the following vessels does the vascular pedicle for this flap originate?

A) Anterior tibial
B) Descending genicular
C) Peroneal
D) Popliteal
E) Posterior tibial

A

The correct response is Option D.

The vascular pedicle for the medial sural artery perforator flap arises from the popliteal vessels.

The medial sural artery flap is a thin, pliable perforator flap that can provide well vascularized soft-tissue coverage, especially for relatively small defects. It is commonly used for head/neck, hand, and lower-extremity defects. The first perforator is frequently found along a line connecting the mid-popliteal area to the medial malleolus at the 8-cm mark from the proximal end. Preoperative planning is facilitated with ultrasound identification of the perforators. Sub-fascial dissection is frequently performed to protect the perforator and blood supply and to allow for a gliding surface for tendon repairs. Donor sites that are narrower than 5 cm can frequently be closed primarily. The main benefit of the medial sural artery perforator flap over an anterolateral thigh flap is the relative thinness of the flap, which can be significant in overweight or obese patients.

How well did you know this?
1
Not at all
2
3
4
5
Perfectly
25
Q

A 24-year-old man comes to the office because of an open wound and osteomyelitis of the right elbow after sustaining a fracture of the olecranon during a fall 1 month ago. Use of the lateral arm flap for coverage of the defect is planned. Which of the following is the arterial supply to the lateral arm flap?

A) Medial collateral
B) Posterior radial collateral
C) Posterior ulnar collateral
D) Radial
E) Ulnar

A

The correct response is Option B.

The posterior radial collateral artery is a branch from the profunda brachial artery, which is off the brachial artery. A second branch is the anterior radial collateral artery but this is variable and of small caliber so does not contribute to the vascular supply. The posterior radial collateral artery interconnects with the radial recurrent artery off the radial artery. This will allow for reverse pedicle design. The middle or medial collateral artery is a branch off the posterior radial collateral artery in 61.5% and off the profunda brachial artery in 38.5%. It can be used as an elongated lateral flap by converting a Y to a V.

The radial, ulnar, and posterior ulnar collateral arteries are not appropriate. The radial artery supplies the radial forearm flap, a fasciocutaneous flap. The ulnar artery supplies a fasciocutaneous flap as well. The posterior ulnar recurrent artery supplies the flexor carpi ulnaris flap, which is a muscle or musculocutaneous flap.

How well did you know this?
1
Not at all
2
3
4
5
Perfectly
26
Q

A 73-year-old man comes to the office for evaluation of an 8-cm mandibular defect with commensurate skin loss 6 weeks after sustaining a gunshot wound to the face. Reconstruction with a free fibula composite flap with skin paddle is planned. Which of the following arteries is the most common origin for blood supply to the skin paddle?

A) Anterior tibial
B) Peroneal
C) Popliteal
D) Posterior tibial
E) Sural

A

The correct response is Option B.

The skin paddle of the free fibula flap receives its vascular supply from the peroneal, posterior tibial vessels, or from both. While a majority (95.8%) of the skin paddles receive their blood supply from the peroneal septocutaneous perforators, a few receive vascular contribution from both peroneal and posterior tibial systems, a few from only the posterior tibial system, and finally, a few from the popliteal artery.

The anterior tibial and sural arteries do not typically contribute to the skin paddle of the free fibula graft.

How well did you know this?
1
Not at all
2
3
4
5
Perfectly
27
Q

A 57-year-old man comes to the office because of a rectourethral fistula that developed after he underwent radiation treatment for prostate cancer. Reconstruction with a pedicled muscle-only gracilis flap is performed. From which of the following directions does the medial femoral circumflex artery pedicle enter the gracilis muscle?

A) Anterior
B) Inferior
C) Lateral
D) Medial

A

The correct response is Option C.

The gracilis muscle is a useful flap for perineal reconstruction. It was first described for use in rectourethral fistula repair by Ryan et al. in 1979. The gracilis muscle is the most superficial of the adductor group and can easily be found in the mid thigh, traversing between the pubic tubercle and medial femoral condyle. Its blood supply is from the profunda femoris as a direct branch or terminal branch of the medial femoral circumflex. There are multiple additional minor pedicles along the muscle’s length (Mathes and Nahai type II). The dominant pedicle enters the muscle approximately a handbreadth below the inguinal crease. It enters the deep aspect of the muscle (ie, from lateral to medial) making dissection of the superficial muscle safe and easy.

How well did you know this?
1
Not at all
2
3
4
5
Perfectly
28
Q

A 45-year-old man is evaluated for unstable plantar scar 3 years after undergoing skin grafting for a traumatic amputation at the tarsometatarsal joints. A photograph is shown. An anterolateral thigh flap is planned for coverage of the resultant plantar defect. Which of the following coaptations is most likely to allow for sensory recovery of the flap?

A) Lateral femoral cutaneous nerve to a deep peroneal nerve branch
B) Lateral femoral cutaneous nerve to a superficial peroneal nerve branch
C) Lateral femoral cutaneous nerve to a tibial nerve branch
D) Medial femoral cutaneous nerve to a deep peroneal nerve branch
E) Medial femoral cutaneous nerve to a tibial nerve branch

A

The correct response is Option C.

The medial femoral cutaneous nerve provides sensation to the anteromedial, not the anterolateral, thigh flap.

Achieving durable results after reconstruction of defects on the weight-bearing surface of the foot is challenging for two main reasons: flap donor sites (other than the medial plantar artery flap) do not have the specialized skin structures of the sole of the foot and are thus less durable than native foot skin; a transferred flap will always be less sensate than native, uninjured plantar foot skin. Flaps are thus more vulnerable to trauma because they cannot feel, and they are less able to tolerate trauma because they lack the native characteristics of plantar skin.

Coapting the sensory nerve of a flap to the native sensory nerve of the recipient area will allow a flap to recover some sensibility, and thus it may be more able to tolerate weight bearing. The sensory innervation to the anterolateral thigh flap is the lateral femoral cutaneous nerve. The sensory innervation to the plantar midfoot is the medial plantar nerve, a terminal branch of the tibial nerve.

The superficial peroneal nerve provides sensation to the dorsal foot.

The deep peroneal nerve provides sensation to the dorsal foot.

How well did you know this?
1
Not at all
2
3
4
5
Perfectly
29
Q

A 58-year-old man undergoes a left hemimandibulectomy. Reconstruction with an osteocutaneous free flap harvested from the ipsilateral pelvis is planned. The vascular pedicle supplying this flap is based on which of the following arteries?

A) Ascending branch of the lateral circumflex femoral artery
B) Deep circumflex iliac artery
C) Deep inferior epigastric artery
D) Descending branch of the geniculate artery
E) Peroneal artery

A

The correct response is Option B.

The deep circumflex iliac artery arises from the external iliac artery and is the blood supply to the iliac crest osteocutaneous flap. This flap can be harvested either as a bone-only or an osteocutaneous free flap. It is often used in hemimandibular reconstruction because the natural curvature of the iliac crest closely resembles the shape of the hemimandible. A portion of the internal oblique muscle, based on the ascending branch of the deep circumflex iliac artery, can also be included with this flap.

The descending branch of the geniculate artery is the blood supply to the medial femoral condyle flap. The ascending branch of the lateral circumflex femoral artery is the blood supply to the tensor fascia lata flap. The peroneal artery is the blood supply to the fibula flap. The deep inferior epigastric artery is the blood supply to the rectus abdominis myocutaneous flap.

How well did you know this?
1
Not at all
2
3
4
5
Perfectly
30
Q

A 50-year-old man is scheduled to undergo mandibulectomy with floor of mouth resection for cancer. An osteocutaneous radial forearm free flap is being considered. Which of the following is the strongest contraindication to performing this flap?

A) Abnormal Allen test
B) Anterior mandibular tumor location
C) Defect length of 8 cm
D) History of prior radiation
E) Lack of availability of ipsilateral neck muscles as recipients

A

The correct response is Option A.

The osteocutaneous radial forearm free flap (OCRFFF) is based on the radial artery, one of the major sources of blood to the hand. An abnormal Allen test is a sign of insufficient ulnar artery blood flow and would be a major contraindication to utilizing this flap.

While the quantity of bone available for transfer from the radius is thought of as a limitation of this flap, many authors have reported safely harvesting up to 10 cm or more of bone length. The thickness of the bone is also a limitation and it is recommended that no more than one-third to one-half of the bone thickness be harvested to avoid an iatrogenic radial fracture, even when the remaining bone is prophylactically plated and/or bone grafted. Because of this, osseointegrated implants for dental restoration can rarely be performed. The bone component of the OCRFFF is well vascularized and associated with high rates of union and can tolerate osteotomies needed for anterior mandible restoration as well as be used for closing irradiated wounds. An advantage of this flap, in addition to providing a thin, pliable skin paddle, is that it has a long pedicle length that can often reach the inspilateral transverse cervical blood vessels or contralateral neck blood vessels without the need for interposition vein grafting.

How well did you know this?
1
Not at all
2
3
4
5
Perfectly
31
Q

A 35-year-old paraplegic man presents with a 5 × 5-cm pressure ulcer over his left ischium involving the skin, subcutaneous tissue, and bone. After debridement, a gluteal rotation flap is planned for closure. Compared with a transposition flap, which of the following is the most significant benefit of using a rotation flap for coverage of this patient’s wound?

A) Ability to reuse flap for future surgery
B) Decreased recurrence rate
C) Improved pressure relief
D) Improved scar placement
E) Improved vascularity

A

The correct response is Option A.

Ischial pressure ulcers can be one of the most difficult wounds for which to achieve long-term coverage and success. The best outcomes arise from multidisciplinary care teams that focus on pressure alleviation (both perioperative and chronic), nutrition, smoking cessation, muscle spasm management, and vigilant observation. Despite best efforts, ischial pressure ulcers still have up to a 70% recurrence rate. Because of high recurrence rates, surgical planning needs take potential future wounds into consideration. Large rotation or advancement flaps have the benefit of being able to be re-rotated or re-advanced, whereas transposition flaps do not, and can make future surgery more difficult. With good surgical planning, rotation and transposition flaps would not have a difference in vascularity, scar placement, pressure relief, or recurrence rates.

How well did you know this?
1
Not at all
2
3
4
5
Perfectly
32
Q

A 53-year-old woman comes to the office with an exposed vascular graft in the left groin caused by postoperative wound infection. After operative debridement, a proximally based sartorius muscle flap is transferred to cover the graft. Several days after surgery, it is evident that the distal portion of the muscle flap is necrotic. This most likely occurred because the sartorius muscle demonstrates which of the following Mathes-Nahai types of vascular pattern?

A)
B)
C)
D)
E)

A

The correct response is Option D.

The sartorius muscle demonstrates a Mathes-Nahai Type IV vascular pattern with multiple segmental vascular pedicles. For this reason, it is important to preserve as many segmental pedicles as possible when transferring this flap. The sartorius flap is usually transferred as a distally based flap, and not a proximally based flap as presented in the clinical scenario. When based proximally, it is necessary to ligate multiple segmental pedicles to achieve an adequate arc of rotation, and this can lead to flap necrosis. When based distally, it is often necessary to divide 1 to 2 segmental pedicles superiorly to allow an adequate arc of rotation, but this usually does not compromise the flap. If there is any concern regarding flap perfusion, the pedicles can be temporarily clamped before division to assess the effects of pedicle ligation.

Examples of muscle flaps with the other Mathes-Nahai types of vascular pattern include the following:
Type I – tensor fascia lata
Type II – gracilis
Type III – gluteus maximus
Type V – latissimus dorsi

How well did you know this?
1
Not at all
2
3
4
5
Perfectly
33
Q

A 45-year-old man is brought to the emergency department 2 hours after sustaining an avulsion injury to the dorsum of the left hand in a motorcycle accident. Physical examination shows complete loss of the dorsal skin and exposure of the extensor tendons in the dorsal hand with no viable peritenon. Reconstruction of the defect with a lateral arm free flap is planned. Which of the following arterial pedicles will supply this flap?

A) Anterior interosseous
B) Inferior cubital cutaneous
C) Posterior radial collateral
D) Radial recurrent
E) Superior ulnar collateral

A

The correct response is Option C.

The correct answer is the posterior radial collateral artery, which is a branch of the profundus brachial artery. The profundus brachial artery arises from the brachial artery and accompanies the radial nerve. The posterior radial collateral artery passes posterior to the lateral intramuscular septum between the deltoid tubercle and epicondyle. The flap can be harvested from the same extremity, offering pliable tissue for soft-tissue coverage. The lateral arm flap also may be taken with muscle, bone, or cutaneous nerves for composite tissue reconstruction.

The anterior interosseous artery supplies the posterior interosseous artery flap (via its connection to the posterior interosseous artery through the distal interosseous membrane) located on the dorsum of the forearm. This flap may be reversed to cover small to moderate defects on the dorsum of the hand.

The superior ulnar collateral artery arises from the brachial artery in the upper arm. This artery is the blood supply of the medial arm flap. The medial arm flap offers the advantage of a well-hidden donor site; however, flap dissection may be extremely tedious and the flap may have significant subcutaneous fat.

The inferior cubital cutaneous artery may be used as a source of a fascial cutaneous flap in the forearm.

The radial recurrent artery arises from the radial artery below the elbow. This artery anastomoses with the radial collateral artery and can be used as a reversed pedicle flap of the lateral arm skin. This flap may reach to the level of the mid-forearm but cannot reach the hand.

How well did you know this?
1
Not at all
2
3
4
5
Perfectly
34
Q

A 45-year-old man is evaluated because of a traumatic plantar heel wound following a calcaneal fracture of the left foot. A pedicled fasciocutaneous flap from the plantar instep is designed for reconstruction. Which of the following best describes the anatomic location of the arterial pedicle of this flap?

A) Between the abductor hallucis and flexor digitorum brevis
B) Between the flexor digitorum brevis and abductor digiti minimi
C) Between the flexor hallucis brevis and adductor hallucis
D) Between the flexor hallucis longus and quadratus plantae
E) Between the tendons of the extensor hallucis longus and extensor digitorum longus

A

The correct response is Option A.

Reconstruction of the weightbearing plantar surface ideally requires skin that is sensate and glabrous. When possible, replacing like-with-like tissue is preferred. The donor site for the medial plantar artery flap is located on the non-weightbearing plantar surface and provides tissue that is structurally similar to the plantar area of the hind foot including fibro-fatty subcutaneous tissue and plantar fascia.

The medial plantar artery is a terminal branch of the posterior tibial artery, and lies between the abductor hallucis and flexor digitorum brevis. Fibers of the medial plantar nerve can be harvested with the flap to provide sensation. The flap can also be raised as a distally-based flap from retrograde flow through the lateral plantar artery for forefoot wounds, or as a free flap. This flap has been shown to provide reliable reconstruction of the plantar surfaces.

The lateral plantar artery runs between the flexor digitorum brevis and abductor digiti minimi. The dorsalis pedis artery runs between the extensor hallucis longus and extensor digitorum longus tendons. The remaining muscle intervals do not contain any major arterial branches used in flap reconstruction.

How well did you know this?
1
Not at all
2
3
4
5
Perfectly
35
Q

Which of the following arteries most likely supplies a vascularized bone free flap from the medial condyle of the femur?

A) Descending genicular
B) Medial femoral circumflex
C) Posterior radial collateral
D) Posterior tibial
E) Sural artery

A

The correct response is Option A.

There has been recent increased interest in the use of the medial condyle of the femur as a source of vascularized bone. The blood supply appears to be robust and predictable. In one study, the descending genicular artery was present in 89%, and the superior medial genicular artery was present in 100% of specimens with average distances proximal to the articular surface of 13.7 cm and 5.2 cm, respectively.

The posterior radial collateral artery is the pedicle for the lateral arm flap. The sural artery perfuses the posterior skin of the calf. The medial femoral circumflex artery lies in the upper thigh and helps supply blood to the neck of the femur. The posterior tibial artery carries blood to the plantar surface of the foot from the posterior artery.

How well did you know this?
1
Not at all
2
3
4
5
Perfectly
36
Q

A 14-year-old girl who has flexion deformities of the right wrist and fingers, numbness of the radial-volar hand, and forearm atrophy 6 months after sustaining an electrical burn to the right upper extremity is brought for evaluation. A functional free gracilis muscle transfer is planned to improve finger flexion. Which of the following nerves innervates this flap?

A) Femoral
B) Genitofemoral
C) Ilioinguinal
D) Obturator
E) Pudendal

A

The correct response is Option D.

The gracilis muscle is the workhorse for functional muscle transfer, and it has been successfully used to restore hand function in patients with severe Volkmann ischemic contracture. Innervation of the muscle is via a branch of the obturator nerve, which is composed of 2 to 3 fascicular bundles. The nerve length from its emergence from the obturator foramen to its insertion into the muscle averages 7.7 cm, and separation of the fascicular bundles can allow the muscle to be segmented. The other nerves listed do not innervate the gracilis muscle.

How well did you know this?
1
Not at all
2
3
4
5
Perfectly
37
Q

A 26-year-old woman is evaluated for burns on the dorsum of the right hand. After serial debridement is performed, the patient has exposed extensor tendons, wrist capsule, and superficial radial nerve. A pedicled groin flap is planned for coverage. Which of the following vessels is most likely to supply arterial blood to the groin flap?

A) Deep circumflex iliac
B) Deep external pudendal
C) Lateral femoral circumflex
D) Superficial circumflex iliac
E) Superficial inferior epigastric

A

The correct response is Option D.

Although all of these vessels supply flaps in the region of the lower abdominal wall, groin, and thigh, the superficial circumflex iliac artery (SCIA) is the dominant pedicle for free or pedicled, fasciocutaneous groin flaps. The long axis of the flap is centered over a line parallel and 3 cm inferior to the inguinal ligament, with a maximum reliable width of 10 cm. The flap is harvested from lateral to medial, beginning caudad to the posterior iliac spine and extending across the sartorius muscle to femoral vessels. To improve the reliability of the flap, Scarpa’s fascia should be incorporated with the flap laterally, and dissection must continue below the sartorius fascia, as the deep circumflex iliac artery (DCIA) courses between the deep investing fascia of the sartorius and Scarpa’s fascia. The base of the flap can be tubed to improve wound care and allow for closure of most of the donor site.

How well did you know this?
1
Not at all
2
3
4
5
Perfectly
38
Q

A 42-year-old man undergoes resection of a dermatofibrosarcoma protuberans of the lower abdomen. A 15 × 9-cm skin and subcutaneous defect results and is closed with a pedicled anterolateral thigh flap. The most common dominant blood supply to this flap is which of the following arteries?

A) Ascending branch of the lateral femoral circumflex
B) Ascending branch of the medial femoral circumflex
C) Descending branch of the lateral femoral circumflex
D) Descending branch of the medial femoral circumflex
E) Transverse branch of the lateral femoral circumflex

A

The correct response is Option C.

The anterolateral thigh flap (ATL) is most frequently used as a free flap. However, it is an extremely reliable and versatile pedicled flap which can be used to reconstruct a variety of lower abdominal, perineal, and pelvic defects. Proximally based ATL flaps are based on flow from the descending branch of the lateral femoral circumflex artery. The descending branch runs between the vastus lateralis laterally and the rectus femoris medially. The perforators from this vessel run through the vastus lateralis to supply the overlying skin. Perforators usually penetrate the anterior aspect of the vastus lateralis. In less than 15% of cases, the perforators run in the septum between the vastus lateralis and the rectus femoris, and in this case, no intramuscular dissection is required.

The medial femoral circumflex is the other main branch of the profunda femoris artery. It divides into the ascending superficial, deep, and acetabular branches and supplies the adductor brevis and magnus as well as the femoral neck. The lateral femoral circumflex divides into ascending, descending, and transverse branches. The ascending or transverse branch and its perforators supplies the tensor fascia lata muscle, and the descending branch supplies the rectus femoris. Its perforators supply the vastus lateralis and its overlying skin.

How well did you know this?
1
Not at all
2
3
4
5
Perfectly
39
Q

A 45-year-old man is evaluated 2 weeks after open reduction and internal fixation of an extra-articular fracture of the distal tibia because of wound dehiscence. Physical examination shows exposed hardware and a 5 × 3-cm open wound above the medial malleolus. After thorough debridement, hardware removal, and placement of an external fixator, the wound is closed with a perforator propeller flap based on a posterior tibial artery perforator. During dissection and inset, the flap becomes progressively swollen and blue, with brisk capillary refill noted for the entire length of the flap. Which of the following is the most appropriate next step in management?

A) Application of nitropaste
B) Free flap salvage
C) Postoperative leech therapy
D) Proximal perforator dissection to source vessel
E) Observation

A

The correct response is Option D.

This patient has an open wound in the distal third of the tibia closed with a propeller flap based on a perforator from the posterior tibial artery. The most common complications associated with this type of reconstruction are venous congestion and partial flap loss. If congestion is encountered during flap elevation, it is essential to dissect the perforator all the way back to the named source vessel to release all fascial attachments and any areas of potential constriction or tethering. This would be the first strategy to improve venous outflow during flap dissection. The venae comitantes accompanying the perforator are very thin-walled and susceptible to kinking, especially when flap inset requires a 180-degree rotation.

Propeller flaps are island fasciocutaneous flaps based on a single dissected perforator. They are termed “freestyle” because the design of the flap is determined intraoperatively based on the dissection of the perforator, which has variable anatomy. Ideal perforators are greater than 0.5 mm in diameter and pulsatile. Perforator flaps of the lower extremity should be designed longitudinally based on directional blood flow. Most published series report greater than 90% flap survival with an 8 to 10% rate of complications. If tension or kinking still exists after proximal perforator dissection, microsurgical venous supercharging would be the next course of action, if possible. De-rotating the flap and placing it back into the donor site as a delay procedure can be used as a last resort; however, vascular delay is traditionally used to augment the arterial inflow of a flap by allowing the choke vessels to open and organize the flow in an axial fashion. The flap remains susceptible to venous congestion when it is rotated for inset after the delay as well.

A small amount of decreased venous drainage is expected with most perforator flaps; however, it is generally mild, not progressive, and confined to the distal tip of the flap. This venous insufficiency can be observed and will generally resolve in the early postoperative period. In this case, the congestion was noted early during flap dissection along the full length of the flap. For severe early venous congestion, observation is not appropriate.

Leech therapy is often used to relieve venous congestion of flaps and replanted parts postoperatively if additional venous outflow cannot be established surgically. Abandoning further surgical efforts to plan for leeching would not be advised as an initial course of action.

The traditional reconstructive algorithm for reconstruction of distal one-third defects recommends free tissue transfer. The advent of perforator propeller flaps allows for reconstruction with available like tissue that does not involve microsurgery. Free flap reconstruction would generally be the plan following complete flap loss, but it would not be the next step in management of venous congestion.

Acute venous insufficiency should not be managed with nitropaste therapy.

How well did you know this?
1
Not at all
2
3
4
5
Perfectly
40
Q

A 43-year-old electrician sustains a high-voltage electrical injury and undergoes multiple debridement procedures of the right upper extremity. The hand, ulnar aspect of the forearm, and medial upper arm are spared. Two weeks following the injury, a final debridement is performed leaving a 6-cm segment of the brachial artery and median nerve exposed in the proximal forearm. Which of the following is the most appropriate method for wound coverage?

A) Above-elbow amputation
B) Dermal substitute followed by skin graft
C) Free tissue transfer
D) Local tissue flap
E) Split-thickness skin graft

A

The correct response is Option D.

The most appropriate method for wound coverage is a local tissue flap, which could come from the intact medial upper arm and/or ulnar aspect of the forearm. A split-thickness skin graft is not appropriate coverage for vital structures. The time it takes for a dermal substitute to vascularize and form the basis of subsequent grafting is too long to leave such vital structures exposed. Free tissue transfer is an option; however, this patient is 2 weeks out from injury and the associated hypercoagulable state is a relative contraindication if local tissues are available. Above-elbow amputation is not an appropriate option as the hand is spared and there are viable coverage options for this young manual laborer.

How well did you know this?
1
Not at all
2
3
4
5
Perfectly
41
Q

A 62-year-old man is diagnosed with osteosarcoma involving the mandible. Microsurgical reconstruction with a free osseocutaneous flap using iliac bone is planned. The vascular pedicle to this flap is which of the following?

A) Deep circumflex iliac vessels
B) Deep inferior epigastric vessels
C) Superficial circumflex iliac vessels
D) Superficial femoral vessels
E) Superficial inferior epigastric vessels

A

The correct response is Option A.

The deep circumflex iliac artery (DCIA) arises from the lateral aspect of the external iliac artery. From its takeoff point, it travels toward the anterior superior iliac spine (ASIS) between the transversalis fascia and transversus abdominis muscle. Just medial to the ASIS, it gives off an ascending branch which supplies the internal oblique muscle. Lateral to the ascending branch, the DCIA courses through the transversalis fascia along the inner lip of the iliac crest, where it lies in the line of fusion between the iliacus and transversalis fascia, and supplies the iliac crest bone.

The deep inferior epigastric vessels supply transverse rectus abdominis myocutaneous (TRAM) and deep inferior epigastric artery perforator (DIEP) flaps. The superficial circumflex iliac vessels supply the groin flap. The superficial inferior epigastric vessels supply the superficial inferior epigastric artery (SIEA) flap, which comprises the skin and subcutaneous tissue only of the lower ipsilateral hemi-abdomen. The superficial femoral vessels supply flaps such as the sartorius muscle flap.

How well did you know this?
1
Not at all
2
3
4
5
Perfectly
42
Q

A 32-year-old male athlete sustains a contact burn to the right foot. Serial debridement results in exposure of the medial aspect of the first metatarsophalangeal joint. A photograph is shown. Which of the following is the most appropriate option for definitive wound management?

A) Amputation of the great toe
B) Bony debridement and primary closure
C) Coverage with a fasciocutaneous free flap
D) Local tissue rearrangement
E) Negative pressure wound therapy

A

The correct response is Option C.

The most appropriate option to obtain definitive wound coverage is a fasciocutaneous free flap harvested from outside the zone of injury. Amputation is not indicated when the majority of the great toe is viable. In addition, this would be highly morbid for this young athlete. Negative pressure wound therapy alone would promote healing by secondary intention, but with an exposed joint this would likely result in an unstable wound. Local tissue rearrangement in this area results in marked donor site morbidity. Bony debridement and primary closure may lead to a healed wound, but functional morbidity would be high in this athlete.

How well did you know this?
1
Not at all
2
3
4
5
Perfectly
43
Q

A 65-year-old woman has a draining sinus tract at the lower chest 2 weeks after undergoing a cardiac bypass procedure. After extensive debridement, there is a large central defect requiring an omental flap for obliteration of the dead space. Which of the following arteries supplies the omental flap?

A) Gastroduodenal
B) Gastroepiploic
C) Left gastric
D) Superior epigastric
E) Superior mesenteric

A

The correct response is Option B.

The blood supply to the omental flap is through the right and left gastroepiploic arteries.

Understanding the anatomy and blood supply to the omentum is crucial for success in omental flap transfer. The greater omentum is harvested from the transverse colon, as the short gastric vessels are ligated and the gastroepiploic vessels preserved. The omentum can be transposed to the chest through either an opening in the diaphragm or a fascial defect in the abdominal wall.

The left gastric vessels arise from the celiac vessels and supply the lesser curvature of the stomach. The gastroduodenal artery arises from the celiac trunk and provides blood supply to the pylorus and proximal duodenum. One of the terminal branches of the gastroduodenal artery is the right gastroepiploic artery. The superior epigastric artery supplies the rectus abdominis muscle and is not intraperitoneal. The superior mesenteric artery arises from the aorta below the celiac trunk and supplies the lower duodenum through the transverse colon; it does not carry the blood supply necessary for design of an omental flap.

How well did you know this?
1
Not at all
2
3
4
5
Perfectly
44
Q

A 50-year-old woman has wound breakdown in the lumbosacral region after spinal instrumentation, as shown in the photograph on the left. The superior aspect is closed with local paraspinal muscle advancement. The lower aspect is closed with a musculocutaneous V-Y advancement flap, as shown in the photograph on the right. Which of the following Mathes/Nahai classifications is most appropriate for this flap?

A) Type I
B) Type II
C) Type III
D) Type IV
E) Type V

A

The correct response is Option C.

The Mathes and Nahai classification system is useful for predicting clinical applicability of various muscle and musculocutaneous flaps. With proper knowledge of the location and variation of muscle blood supply, the surgeon can safely determine the extent of muscle transposition during surgery. Five patterns of muscle circulation have been described. These patterns are based on the following relationships between the muscle and its vascular pedicle:

The regional source of the vascular pedicle(s) entering the muscle

Pedicle size

Number of vascular pedicles

Location of the pedicle in relation to muscle origin and insertion

The angiographic patterns of intramuscular vessels

The gluteal V-Y advancement flap used in the clinical scenario described is a Mathes/Nahai Type III flap. Type III muscle flaps demonstrate two large, independent vascular pedicles arising from separate regional arteries. Other Type III muscles include the rectus abdominis and serratus anterior. Angiographic studies have shown equal filling of the intramuscular vascular system with either pedicle injection. Type III muscle flaps can be based on either pedicle and can be split to preserve muscle function. In this particular ambulatory patient, only the superior half of the gluteal muscle (based on the superior gluteal artery) was used in order to preserve lower gluteal function.

Type I muscles have a single dominant pedicle. Examples include the gastrocnemius, rectus femoris, and tensor fascia lata flaps.

Type II muscle flaps demonstrate one or more large vascular pedicles near the muscle origin and several small pedicles entering the muscle belly distally. Commonly used muscle flaps in this group include the gracilis, soleus, and trapezius. The minor pedicles are typically divided to allow maximal muscle transposition. Division of the minor pedicles typically has little effect on muscle flap survival, but poorly planned musculocutaneous flaps may suffer distal skin ischemia if not planned appropriately. This vascular pattern is the most common pattern observed in anatomical studies of human cadaveric muscle.

Type IV muscles demonstrate segmental vascularization along the entire length of the muscle. The sartorius and tibialis anterior muscles are the most clinically relevant muscles that display this type of pattern. The segmental nature of the blood supply severely limits the ability to transpose these muscles and therefore the utility is limited.

Type V muscles display one dominant vascular pedicle near the muscle origin and multiple segmental pedicles near the muscle insertion. The latissimus and pectoralis major muscles demonstrate this vascular pattern. Angiographic studies demonstrate that the intramuscular vasculature can be supplied by either the dominant or segmental pedicles. As a result, the flaps can be elevated on either vascular system.

How well did you know this?
1
Not at all
2
3
4
5
Perfectly
45
Q

A 27-year-old woman is evaluated for a traumatic wound to the left heel that she sustained when she was attacked by a shark 6 days ago. Physical examination shows a 5 × 5-cm soft-tissue defect with exposed bone. Reconstruction with a sensate flap taken from the instep region is planned. Which of the following nerves provides innervation to this flap?

A) Calcaneal
B) Lateral femoral
C) Medial plantar
D) Superficial peroneal
E) Sural

A

The correct response is Option C.

The nerve supply to the medial plantar artery flap is the medial plantar nerve.

The medial plantar artery flap is a sensate flap that can be used for coverage of heel defects. The flap incorporates tissue from the medial instep of the foot in a non–weight-bearing area, and can be transposed posteriorly to allow for coverage of heel defects. This flap provides durable plantar glabrous skin that can allow for weight bearing.

The flap is based on the medial plantar artery, which arises from the posterior tibial artery. A branch of the medial plantar nerve providing sensation to the instep of the foot can be harvested with the flap, allowing for preservation of sensation in the reconstructed heel.

The medial plantar artery flap can also be raised as a sensate free flap, with anastomosis of the medial plantar nerve to a nerve at the recipient site.

The calcaneal nerve provides native sensory innervation to the heel.

The lateral femoral cutaneous nerve provides sensation to the anterolateral thigh flap.

The superficial peroneal nerve provides motor innervation to the lateral leg.

The sural nerve provides sensation to the lateral side of the foot.

How well did you know this?
1
Not at all
2
3
4
5
Perfectly
46
Q

A 52-year-old man with a 10-year history of intravenous drug use is evaluated for right groin pseudoaneurysm. He has a history of right groin reconstruction with a local sartorius flap. After vascular reconstruction, he is left with an exposed vascular conduit in need of stable coverage. The local groin tissues are indurated and stiff. The rectus femoris muscle is rotated to provide coverage. Which of the following lower extremity function deficits is most likely in this patient?

A) Inability to abduct the thigh
B) Inability to extend the knee
C) Inability to flex the ipsilateral hip
D) Weakened extension of the knee
E) No functional deficit

A

The correct response is Option D.

The rectus femoris muscle is the most superficial and central of the quadriceps extensor muscle group. It is a bipennate muscle that extends from the ilium to the patella and is surrounded by the vastus lateralis and vastus medialis muscles. The rectus femoris acts as an extensor of the knee, specifically powering the terminal 15 to 20 degrees of knee extension. It is also a powerful flexor of the hip. Recent studies have shown that the use of this muscle is effective for groin reconstruction. Mild, isolated deficits are observed although there is no clinical significance of this deficit.

How well did you know this?
1
Not at all
2
3
4
5
Perfectly
47
Q

A 25-year-old man is brought to the emergency department after he sustained a mutilating injury to the right hand and wrist that requires soft-tissue reconstruction. Examination shows exposed tendon and bone over the dorsum of the right hand and wrist. The zone of injury extends to the level of the elbow. Allen test is abnormal. Which of the following is the most appropriate method of reconstruction?

A) Coverage with a groin flap
B) Coverage with a reverse radial forearm flap
C) Full-thickness skin grafting
D) Negative pressure wound therapy
E) Split-thickness skin grafting

A

The correct response is Option A.

The patient described has a mutilating injury to the dorsum of the hand and wrist with exposed extensor tendons and metacarpals, which would not be an appropriate bed for a skin graft. Skin grafts survive initially by plasmatic imbibition and then by inosculation from the wound bed. Wounds with extensive exposure of tendons do not provide the potential for in-growth of vascularized tissue to maintain a skin graft. The radial forearm flap cannot be used in this patient because the palmar arch has been injured and the patient does not have communication between the radial and ulnar arterial system such that arterial compromise can occur if the radial artery is transected for the flap. Negative pressure wound therapy can be considered temporarily, but will not provide definitive management of this complex wound.

How well did you know this?
1
Not at all
2
3
4
5
Perfectly
48
Q

A 9-year-old girl is evaluated for scar revision after sustaining a laceration of the left cheek that was repaired in the emergency department 2 years ago. Examination shows a 4 × 0.7-cm scar that is pale, flat, and wide in appearance. It has an oblique orientation between the oral commissure and zygomatic arch. W-plasty is considered. Which of the following is the most likely outcome of the W-plasty when compared with linear closure in this patient?

A) Decreased initial wound tension
B) Decreased procedural time
C) Increased contracture of the wound
D) Increased removal of healthy tissue

A

The correct response is Option D.

The W-plasty was first described by Borges in 1953. The repair involves a regular pattern of interdigitating triangular advancement flaps. This repair allows for the scar to have an accordion-like effect secondary to the broken line configuration. The W-plasty and geometric broken line closure (GBLC) techniques are commonly used for scar revisions and are best indicated for scars that cross the relaxed skin tension lines (RSTL) to redirect portions of the scar. Because of the flexibility these scars have, they are also indicated over convex or concave surfaces.

Compared with linear closure, the W-plasty will have decreased contracture of the wound because of the interrupted orientation of the scar. Even though a portion of the final closure will go against the RSTLs, W-plasties can be designed to orient a significant portion along the RSTLs. Because of the many triangular flaps made and repaired, it takes significantly longer to perform a W-plasty repair. The main disadvantage of the W-plasty and GBLC procedures is the need to remove a significant amount of healthy tissue on either side of the scar to be revised, which can lead to increased wound tension and the need for significant undermining.

How well did you know this?
1
Not at all
2
3
4
5
Perfectly
49
Q

A 24-year-old right-hand-dominant male construction worker is evaluated because of a right dorsal thumb abscess that is treated with debridement and administration of antibiotics. A photograph of the residual defect is shown. Which of the following is the most appropriate method for reconstruction in this patient?

A ) Coverage with a muscle flap
B ) Coverage with a skin flap
C ) Full-thickness skin grafting
D ) Negative pressure wound therapy
E ) Split-thickness skin grafting

A

The correct response is Option B.

The residual defect includes exposed extensor tendon without paratenon. This fact, combined with the need for flexion at the interphalangeal joint and avoidance of contracture, as well as the likely need for future tenolysis, makes a skin flap the most appropriate option for reconstruction. In the scenario described, a first dorsal metacarpal artery pedicled skin flap is used to reconstruct the thumb defect with the need for back grafting of the donor site. This provides the best combination of low donor-site morbidity, the ability to provide stable soft-tissue coverage over exposed tendon without paratenon, and the competitive advantage of being relatively easy to re-elevate for subsequent procedures, if needed.

A muscle flap could be used to reconstruct the defect but would not be optimal due to the increased donor site morbidity from muscle sacrifice, as well as the increased difficulty in re-elevation versus a skin flap over tendon.

Healing by secondary intention, with or without topical negative pressure wound therapy, will certainly result in extensive contracture as well as an increased time to heal. This will impact the patient’s outcome both in terms of his ability to return to work as a construction worker as well as limitations on his functional range of motion.

Skin grafting, whether split- or full-thickness, is not a reliable option in this patient because of the exposed tendon without paratenon. It is important to note that this is a classic contraindication to skin grafting and therefore leads to a flap-based reconstruction.

Furthermore, skin grafts would lead to increased contraction versus flaps and would be difficult to re-elevate for subsequent procedures.

50
Q

A healthy 40-year-old man comes to the office because of an injury to his finger. Examination shows a 4 × 3-cm, full-thickness defect over the proximal interphalangeal (PIP) joint of the right index finger. The joint capsule and extensor tendons are exposed. No sign of infection is noted. Coverage with an arterialized venous free flap from the forearm is planned. Which of the following is the main disadvantage of using this flap in reconstruction?

A ) Difficult flap monitoring
B ) High incidence of total flap failure
C ) Inability to transfer flap with nerve and/or tendon
D ) Limited supply of donor sites
E ) Technically demanding flap harvest

A

The correct response is Option A.

Arterialized venous free flaps are thin fasciocutaneous flaps that are useful in reconstructing defects of the hand. The main advantage of an arterialized venous flap is the ease of harvesting a thin flap without the need to sacrifice a major artery at the donor site. The design of venous flaps is very easy because of direct visualization of the venous plexus through the thin overlying skin. There is no limitation of the donor site because it is possible to find the venous network at any location of the body. They can be harvested as composite flaps, including nerve and tendon flaps. Despite many advantages, arterialized venous flaps are not commonly selected as the first choice for microsurgical reconstruction. Venous congestion is an expected part of the flap’s postoperative course, and this makes monitoring the flap especially difficult as compared with other conventional flaps. Signs of vascular insufficiency and characteristic edema and congestion of venous flaps are difficult to differentiate. In cases of venous congestion, flap viability can be monitored only by palpating the pulse or by laser Doppler probe analysis. Though venous congestion is common, the ultimate success rate of arterialized venous flaps is similar to that of conventional flaps, with total flap loss a rare occurrence.

51
Q

An 18-year-old man undergoes open reduction with tension band wiring to treat a fracture of the olecranon he sustained in a bicycle collision. Postoperatively, he develops a wound infection, resulting in an open wound over the elbow. Coverage with a posterior interosseous fasciocutaneous flap is planned. Which of the following best describes the anatomical location of the dominant pedicle of this flap?

A ) Anterior to the pronator teres and deep to the brachioradialis
B ) Between the extensor carpi ulnaris and the extensor digiti minimi
C ) Between the flexor digitorum profundus and flexor pollicis longus
D ) Deep to the brachioradialis and lateral to the flexor carpi radialis
E ) Superficial to the anconeus and extensor digitorum muscle

A

The correct response is Option B.

The posterior interosseous flap is a pedicled forearm flap based off the posterior interosseous artery (PIA) proximally. It can be rotated to cover elbow, antecubital fossa, or proximal volar forearm defects. A reversed version, based off the anterior interosseous arterial connections to the PIA, can be used for wrist and hand defects.

The PIA emerges in the proximal dorsal forearm deep to the supinator. It then courses between the extensor carpi ulnaris (ECU) and the extensor digiti minimi (EDM). During dissection of a standard posterior interosseous flap, the pedicle is found distally in the forearm between the ECU and EDM then dissected proximally.

The superficial branch of the radial nerve is located anterior to the pronator teres and deep to the brachioradialis. The anterior interosseous artery is found between the muscle bellies of flexor digitorum profundus and the flexor pollicis longus.

Distally in the forearm, the radial artery is deep to the brachioradialis and radial to the flexor carpi radialis. Proximally in the forearm, the posterior cutaneous nerve of the forearm is found superficial to both the anconeus and extensor digitorum muscle.

52
Q

A 50-year-old woman has significant tenderness of the residual tip of the index finger 8 weeks after undergoing amputation of the fingertip. Physical examination shows significant stump tenderness. Which of the following therapy modalities is most appropriate for desensitization of the amputation stump?

A ) Functional electric stimulation
B ) Immobilization
C ) Kinesiology tape
D ) Semmes-Weinstein monofilaments
E ) Vibration

A

The correct response is Option E.

Vibration is a modality that may be used in therapy to desensitize an amputation stump neuroma. The treatment consists of vibratory stimulation applied to the periphery of the sensitive area and then gradually moving toward the center. Some additional methods used in therapy to treat neuroma pain include desensitization, massage, and transcutaneous nerve stimulation. Functional electric stimulation is similarly not a therapeutic modality. Kinesiology taping is used for edema control and comfort. It would not be used on an amputated stump. Semmes-Weinstein monofilaments are used to quantify sensation to fine touch and are not used for desensitization.

53
Q

A 48-year-old man is brought to the emergency department after being involved in a high-speed motorcycle collision. Physical examination shows comminuted fractures of the left radius and ulna. A photograph of his wounds is shown. After debridement of the necrotic tissue, the brachial vessels and median nerve are exposed. Microsurgical techniques are not available. Which of the following is the most appropriate choice for tissue coverage in this patient?

A ) Groin flap
B ) Lateral arm flap
C ) Posterior interosseous artery flap
D ) Radial forearm flap
E ) Split-thickness skin graft

A

The correct response is Option B.

The vascular supply of the lateral arm flap is from the posterior radial collateral artery, a terminal branch of the deep brachial artery. This artery communicates distally with the ulnar artery, allowing the flap to be raised in a distally based fashion, as it was for the patient in this item. For most adult patients, a 12 × 6-cm flap can be harvested and still allow for primary donor site closure. The major extremity vessels are not disturbed with the harvest of this flap.

A groin flap would be difficult to mobilize sufficiently to reach the defect shown. It would also require leaving the arm attached to the torso for several weeks for vascular ingrowth prior to division and inset.

The posterior interosseous artery flap is based on the communication between the anterior and posterior interosseous arteries just proximal to the distal radioulnar joint. Its pedicle can be used in a distally based fashion to allow for coverage of defects of the hand and wrist. It cannot reach to cover defects at the elbow.

A radial forearm flap has two major disadvantages. It will remove a major vessel to a wrist and hand that is needed to heal a severe injury. It will also expose the radius fracture, requiring secondary flap reconstruction of the donor site. Split-thickness skin grafting is inappropriate to place over an exposed nerve, as sensitivity and pain would be expected to persist after healing. In addition, a skin graft would suffer secondary contraction, which would impair elbow extension.

54
Q

A 28-year-old man is evaluated in the emergency department because of a soft-tissue defect of the dorsum of the hand with exposed extensor tendons. Reconstruction is planned with a fasciocutaneous free flap that is centered on the axis between the anterior superior iliac spine and patella. Which of the following arteries is the most likely pedicle of this flap?

A ) Ascending branch of lateral femoral circumflex
B ) Ascending branch of medial femoral circumflex
C ) Descending branch of lateral femoral circumflex
D ) Descending branch of medial femoral circumflex
E ) Superficial femoral

A

The correct response is Option C.

The pedicle to the anterolateral thigh flap is the descending branch of the lateral femoral circumflex artery. The anterolateral thigh flap is a fasciocutaneous flap that has gained in popularity. This flap is capable of providing pliable tissue with a fascia to allow for tendon gliding. The flap has a large skin territory and does not require the sacrifice of a major vessel.

The flap is based on perforators from the descending branch of the lateral femoral circumflex artery, which arises from the profunda femoris artery. The artery travels between the vastus lateralis and rectus femoris muscles and may travel in the septum or within the substance of the muscles.

The flap is centered on the axis between the anterior superior iliac spine and the superior lateral border of the patella. Perforators to the anterolateral thigh flap can be variable, but the majority lies within 3 cm of a circle centered along the midpoint of that line.

The ascending branch of the lateral femoral circumflex artery forms the pedicle for the tensor fascia lata flap.

The ascending branch of the medial femoral circumflex artery forms the pedicle for the gracilis muscle flap.

The descending branch of the medial femoral circumflex artery supplies the adductor muscles.

55
Q

A 24-year-old man is brought to the emergency department after sustaining a degloving injury of the long, ring, and little fingers of the dorsal, nondominant left hand in a high-speed, rollover motor vehicle collision. Following debridement, the patient has obvious open proximal interphalangeal (PIP) joints of each of these fingers. Which of the following is the most appropriate management of these defects?

A ) Cross-finger flap coverage
B ) Full-thickness skin grafting from the groin
C ) Pedicle lateral arm flap coverage
D ) Reverse radial forearm flap coverage
E ) Split-thickness skin grafting from the thigh

A

The correct response is Option D.

Pedicled fascial flaps to the hand provide an excellent reconstructive option in cases of exposed tendon, joint, or bone where soft-tissue coverage is needed. They provide thin, broad, well-vascularized coverage and a gliding surface for tendons and joints. The pedicled reverse radial forearm flap is the most appropriate option for this defect. The cross-finger flap may be an option for a single digit, but not in the large zone-of-injury described. The groin flap, although reliable, is bulky and requires the attachment of the upper extremity to the trunk, followed by division and insetting of the flap at a later time. The pedicle lateral arm flap cannot reach the fingers. The reverse radial forearm flap may be harvested with skin or simply as a fascial flap. Skin grafting, either full- or split-thickness, would not be appropriate coverage for exposed joint surfaces.

56
Q

A 46-year-old man undergoes resection of a sarcoma involving the posterior neck. Photographs are shown. Which of the following arteries supplies the musculocutaneous flap used to reconstruct this defect?

A ) Circumflex scapular
B ) Deep temporal
C ) Internal carotid

D) Superficial temporal
E ) Transverse cervical

A

The correct response is Option E.

The transverse cervical artery is the blood supply to the trapezius flap which is used to reconstruct the defect described. The circumflex scapular artery is the blood supply to the scapular/parascapular flap. This artery has a length of approximately 6 to 7 cm and does not have the arc of rotation to reach a posterior neck defect. The deep temporal artery is the blood supply of the temporalis muscle. The arc of rotation of this muscle does not enable repair of posterior neck defects. The internal carotid artery does not give rise to branches that supply the muscles of the neck/back that may be useful for reconstruction of this defect. The superficial temporal artery is the blood supply of the superficial temporal fascia. This flap is quite thin with a limited reach and would not be appropriate for reconstruction of an extensive resection that includes skin, subcutaneous tissues, and muscle as shown.

57
Q

A 28-year-old man is brought to the emergency department after sustaining a dog bite to the face. Physical examination shows subtotal loss of the nose and glabella. Staged reconstruction with a forearm flap is performed, with initial elevation of the flap, placement of cartilage grafts, and creation of nostrils. Thinning and refinement are performed during a second procedure with additional cartilage grafting. The flap is microsurgically transferred to reconstruct the nose in a third procedure. Which of the following is the most appropriate description of this flap?

A) Delayed
B) Freestyle
C) Prefabricated
D) Prelaminated
E) Tubularized

A

The correct response is Option D.

The flap described in this scenario is a prelaminated flap. A prelaminated flap is an axial flap that is modified with the addition of various grafts (e.g., skin, mucosa, cartilage, bone), re-creating the missing tissues at the donor site prior to flap transfer.

A delayed flap is one that undergoes one or more vascular insults prior to final flap elevation to induce increased circulation and maximize flap perfusion. A freestyle flap is a nonaxial flap harvested by locating a cutaneous Doppler signal in a chosen donor site, identifying the vessels supplying that tissue, and dissecting them down to a pedicle of sufficient length and/or diameter. The anatomy is not known ahead of time, and thus harvest is performed ?freestyle.? A prefabricated flap is created by transferring a vascular pedicle into an area of tissue that is ideal for transfer to induce angiogenesis from the pedicle into that tissue, which can then be harvested for transfer. A tubularized flap is one that is sewn to itself to create a tube or passive conduit, such as an anterolateral thigh flap used for pharyngoesophageal reconstruction.

58
Q

A 52-year-old man is brought to the emergency department after sustaining injuries in a motor vehicle collision. Physical examination shows a traumatic degloving injury to the dorsum of the right hand with exposed, intact extensor tendons. Reconstruction with a fascial free flap and full-thickness skin grafting are planned. Which of the following arteries supplies blood to the most appropriate choice of flap?

A) Posterior auricular
B) Superficial temporal
C) Superior thyroid
D) Supratrochlear
E) Transverse facial

A

The correct response is Option B.

The temporoparietal fascial flap is supplied by the superficial temporal artery. This thin fascial free flap is useful in reconstruction of traumatic injuries that are not amenable to reconstruction with a skin graft alone. This flap is particularly useful in reconstruction of gliding surfaces with denuded tendons or exposed joints.

The posterior auricular artery is a branch of the external carotid artery and supplies the posterior ear. The superior thyroid artery arises from the external carotid artery and supplies the thyroid gland. The supratrochlear artery supplies the forehead and scalp. The transverse facial artery is a branch of the superficial temporal artery and supplies the parotid gland. None of these vessels is involved in the vascular supply of the temporoparietal fascial flap.

59
Q

A 48-year-old man has infected hardware 4 weeks after undergoing spinal fusion. The neurosurgeon washes out the wound and requests consultation for coverage of the defect. In the operating room, the plastic surgeon notes that coverage with a paraspinous muscle flap is not possible, as the muscle has been heavily debrided by the neurosurgeon. Use of a reverse latissimus dorsi flap is planned. These two flaps share an arterial blood supply from which of the following arteries?

A) Circumflex scapular
B) Posterior intercostal
C) Superior gluteal
D) Thoracodorsal
E) Transverse cervical

A

The correct response is Option B.

The paraspinous muscle is supplied by the posterior intercostal artery; this is the same vessel that supplies the reverse latissimus dorsi flap. For a midline spinal defect, it is unlikely that these vessels are damaged; however, this is possible in large oncologic resections or traumatic injuries.

The circumflex scapular artery supplies multiple flaps that are used for reconstruction, including the scapular and parascapular flaps. The transverse cervical artery supplies the trapezius muscle flap. The superior gluteal artery supplies the gluteus maximus muscle. The thoracodorsal artery is the main arterial supply of the latissimus muscle.

60
Q

A 55-year-old woman is scheduled to undergo surgery for soft-tissue coverage of an open joint elbow wound. The vascular pedicle of the flap in the photograph shown passes between which of the following tendons?

A) Brachioradialis and abductor pollicis longus
B) Brachioradialis and flexor carpi radialis
C) Brachioradialis and flexor pollicis longus
D) Brachioradialis and pronator teres
E) Flexor carpi radialis and pronator teres

A

The correct response is Option B.

The clinical image shown depicts a radial forearm flap. The vascular pedicle of the radial forearm flap is the radial artery that is a branch of the brachial artery. Proximally, the radial artery runs deep to the brachioradialis muscle and it passes distally between the bellies of the brachioradialis and flexor carpi radialis. The radial forearm flap cutaneous paddle is perfused by septocutaneous perforators from the radial artery.

The other options are incorrect because the vascular pedicle does not pass between those muscles.

References

61
Q

A 45-year-old woman is brought to the emergency department after sustaining a shotgun injury to the nondominant left forearm. A preoperative x-ray study is shown. Bone stabilization is performed. The ulnar nerve, multiple flexor tendons, and the ulnar artery are repaired during surgery and are left exposed. A photograph taken following the repair is shown. The wound was covered with allograft skin while viability of the hand was confirmed for 2 days. Which of the following is the most appropriate definitive coverage for the wound?

A) Free anterolateral thigh flap
B) Full-thickness skin grafting from the groin
C) Pedicled groin flap
D) Reverse lateral arm flap
E) Split-thickness skin grafting from the thigh

A

The correct response is Option A.

Shotgun blasts at close range can create a devastating pattern of injury. X-ray study shows a heavily comminuted ulna fracture. The scenario also involves tendon, vascular, and nerve injury. In choosing the appropriate coverage for the wound described, the surgeon will need to consider the protection of exposed structures, the ability to rehabilitate the extremity, and the possible need for future surgery on the arm.

A free tissue transfer will provide viable, full-thickness tissue from a nontraumatized area to cover the wound. Many donor sites are available, including fasciocutaneous flaps such as the anterolateral thigh flap, as well as muscle flaps. Some authors advocate fasciocutaneous flaps over muscle flaps with the rationale that the fat on the deep surface of the flap will better allow glide of tendons and nerves deep to it; in addition, a fasciocutaneous flap, once healed, can be incised like normal skin for any future surgery that might be necessary in the patient’s arm. A photograph is shown.

Skin grafting, whether split-thickness or full-thickness, carries several liabilities. Both types of grafts provide skin but no subcutaneous tissue. In addition, both heal by adhering to the wound bed. Because the patient described has exposed tendon in the wound bed, skin grafting would likely cause significant tendon adhesions and thus impaired mobility. In addition, adhesion of the skin graft to a nerve, particularly a repaired nerve, carries a risk of chronic pain and nerve dysfunction.

A pedicled groin flap can provide thin, pliable, full-thickness tissue to cover defects of the forearm. However, the flap is done as a two-staged procedure, with the arm tethered to the groin for 2 to 3 weeks to allow vascular ingrowth. In the patient described, having the arm dependent for that period of time would significantly worsen edema and compromise the ability to begin rehabilitation of the extremity.

A reversed lateral arm flap provides full-thickness soft-tissue coverage for defects about the elbow. Its harvest does not require sacrifice of a major artery to the hand. The lateral arm flap would not be able to reach the distal limit of the wound described. In addition, the lateral arm flap donor site can be closed primarily for flaps up to 12 × 6 cm, which is significantly smaller than the wound in the patient described.

62
Q

A 55-year-old man is scheduled to undergo a large oncologic extirpation in the groin. Closure of the resulting defect with a rectus femoris musculocutaneous flap is planned. Which of the following is the most likely functional outcome?

A) 15-Degree extensor lag of the knee
B) 20-Degree flexion contracture of the hip
C) Compromised ability to stand for extended periods
D) Inability to adduct the leg
E) No loss of function

A

The correct response is Option A.

The rectus femoris flap provides reliable and robust soft tissue for coverage of abdominal, groin, and hip defects. Because the rectus femoris is one of the quadriceps muscles and inserts into the patella, its use can have functional consequences. It is generally recommended to perform patellar tendon repair following harvest of the rectus femoris; despite this repair, there can still be about 15 degrees of extensor lag at the knee. Flexion contracture of the hip, difficulty standing for extended periods, and difficulty adducting the leg have not been described with this flap harvest. The expanded rectus femoris flap has several advantages for massive abdominal wall reconstruction. The expanded flap easily can reach the xiphoid, and it has impressive width. The donor site can be closed primarily with an acceptable scar. The muscle remains innervated and functional, which may help prevent bulging. Large or complicated abdominal wall defects caused by recurrent incisional hernias, infections, or tumor resections often require the use of prosthetic mesh, local tissue transposition, or even distant muscle flaps for proper reconstruction. In a series of 12 cases of reconstruction of the abdominal wall using pedicled rectus femoris muscle flaps for wounds resulting after tumor resections, recurrent incisional hernias, and infection, abdominal wall stability and donor site morbidity were examined clinically. Follow-up time ranged from 6 months to 4 years. In all but one patient, a stable abdominal wall could be reconstructed. The loss of true muscular capacity in the quadriceps muscle of the operated leg was 19% compared with the nonoperated leg, but this result was tolerated well.

63
Q

A 70-year-old woman has a circular defect 18 cm in diameter on the parietal aspect of the scalp after excision of squamous cell carcinoma. The pericranium has been removed with the scalp tissue. Adjuvant radiation therapy is planned beginning 4 to 6 weeks after surgery. Which of the following is most appropriate for coverage of the defect?

A) Latissimus dorsi muscle free flap with split-thickness skin graft
B) Primary closure after galeal scoring
C) Split-thickness skin grafting
D) Temporary reconstruction with a split-thickness skin graft followed by a rotation-advancement flap after scalp tissue expansion

A

The correct response is Option A.

The latissimus dorsi muscle free flap with split-thickness skin grafting can be used to reconstruct large scalp defects in a single stage, allowing the patient to proceed with radiation therapy after recovery from surgery.

While temporary reconstruction with a split-thickness skin graft followed by definitive reconstruction with a rotation-advancement flap after tissue expansion is feasible for defects approaching 50% of the scalp surface area, this choice is inappropriate, as skin grafts usually have poor take on bare calvarium devoid of pericranium, particularly when treated with radiation. Radiated tissues are more difficult to expand, and their expansion is associated with a high rate of complications. Primary closure is usually only feasible in scalp defects less than 3 cm in diameter, even with galeal scoring to increase scalp flap length and reduce wound tension. Graft take can be improved by burring the bone down to the bleeding diploic space, but this technique results in unstable bone coverage, particularly in the setting of postoperative radiation.

64
Q

A 79-year-old woman comes to the office because of a large scalp defect following Mohs micrographic surgery for basal cell carcinoma. A photograph of the defect (left) and a rotation flap designed to cover the defect (right) are shown. At the completion of the procedure, a large “dog ear” is noted at the pivot point of the flap. Which of the following is the most appropriate next step in management?

A) Burrow triangle
B) Compression
C) Direct excision
D) Staged advancement
E) Observation

A

The correct response is Option E.

The most appropriate next step in management is observation. A Burrow triangle would not sufficiently address this deformity and is not necessary. Compression and staged advancement are not necessary, as the deformity resolves spontaneously. The temptation to excise should be resisted, as most of these ?dog ears? resolve over time. Photographs immediately after the procedure (left) and 5 months later (right) are shown.

Excision would only increase the length-to-width ratio and compromise blood supply to the flap. If the contour abnormality is unacceptable to the patient after some time has passed, then excision at a later time can be performed. The need for this is exceedingly rare.

65
Q

A 50-year-old man is scheduled to undergo resection of a squamous cell cancer of the right floor of the mouth with invasion into the mandibular body. Composite resection of the right hemimandible and a 2-cm resection of the floor of the mouth followed by immediate reconstruction with a fibula flap are planned, necessitating the use of an osteocutaneous flap. Which of the following best describes the course of the blood supply for the skin paddle of this flap?

A) Musculocutaneus and septocutaneous perforators from the peroneus longus and brevis muscles
B) Musculocutaneus perforators from the flexor hallucis brevis muscle
C) Septocutaneous and musculocutaneus perforators from the anterior tibialis muscle
D) Septocutaneous perforators from the posterior intermuscular septum
E) Septocutaneous perforators from the posterior tibialis muscle

A

The correct response is Option D.

The blood supply for the fibula flap skin paddle used for reconstruction arises from septocutaneous perforators from the peroneal artery that traverse the posterior intermuscular septum. The position of these vessels can be marked before the procedure with a pencil Doppler probe. In the majority of cases, the blood supply to the fibula flap skin paddle is reliable for microsurgical transfer. The skin flap also has blood supply arising from musculocutaneus perforators originating from the soleus and gastrocnemius muscles; however, these vessels are, in most cases, ligated because their dissection is tedious, and they often arise from the posterior tibial artery. The anterior and posterior tibialis muscles do not provide blood supply to the fibula flap skin paddle.

66
Q

Which of the following is the most appropriate Mathes and Nahai classification of the rectus abdominis muscle?

A ) Type I

B ) Type II

C ) Type III

D ) Type IV

E ) Type V

A

The correct response is Option C.

The rectus abdominis muscle is a Mathes and Nahai classification Type III muscle.

The Mathes and Nahai classification of the vascular anatomy of muscles represents an important description in reconstructive surgery. Type I muscles have one vascular pedicle and include the gastrocnemius, rectus femoris, and tensor fascia lata. Type II muscles have a dominant and minor pedicle and include the abductor digiti minimi, abductor hallucis, biceps femoris, flexor digitorum brevis, gracilis, peroneus longus, peroneus brevis, platysma, semitendinosus, soleus, sternocleidomastoid, temporalis, trapezius, and vastus lateralis. Type III muscles have two dominant pedicles and include the gluteus maximus, rectus abdominis, serratus anterior, and semimembranosus. Type IV muscles have multiple segmental pedicles and include the extensor digitorum longus, extensor hallucis longus, flexor digitorum longus, flexor hallucis longus, sartorius, and tibialis anterior. Type V muscles have one dominant and secondary segmental pedicles and include the pectoralis major and latissimus dorsi.

67
Q

A 54-year-old man is scheduled for correction of a defect on the back 1 week after undergoing resection of a 3 × 3-cm recurrent sarcoma. The skin was closed primarily during the procedure, but skin necrosis and wound breakdown occurred. History includes radiation therapy to the spine for soft-tissue sarcoma 2 years ago. Physical examination shows a 5 × 5-cm defect in the mid back at the level of T10 with exposed spinous processes. Which of the following is most appropriate to achieve complete wound closure?

A ) Latissimus dorsi flap

B ) Rectus abdominis free tissue transfer

C ) Split-thickness skin graft

D ) Trapezius muscle flap

E ) Wide undermining with primary reclosure

A

The correct response is Option A.

The latissimus dorsi flap would allow for the mobilization of sufficient skin and muscle to close the defect in the patient described. The latissimus dorsi muscle is a Mathes-Nahai Type V flap, with the main blood supply from the thoracodorsal artery and vein, and secondary segmental pedicles from the posterior intercostal and lumbar perforators. The latissimus dorsi insertion onto the humerus can be divided to provide further mobilization of the flap. To close the defect, the flap can be used as a €œturnover flap, € with its blood supply based medially on the secondary pedicles. It can also be transposed and advanced on its main blood supply. If needed, interposition vein grafts can be used to extend the vascular pedicle. The flap has a very well-defined anatomy with few anatomical variations, and its harvest is fairly straightforward. It has found tremendous use in reconstructing defects of the scalp, lower extremity, breast, and chest wall.

A free rectus abdominis muscle flap could be performed, but recipient vessels in this area are not readily available. There have been reports of utilizing posterior intercostal perforators as recipient vessels for microvascular anastomosis, but this is an option best reserved as a last resort. Split-thickness skin grafting over a previously irradiated wound bed with exposed bone is not likely to heal. A trapezius muscle flap will not reach the level of T10. Wide undermining is a poor choice because this would lead to further devascularization of previously irradiated skin. Reclosure of the skin primarily is likely to occur under significant tension and would be prone to further necrosis and wound breakdown.

68
Q

A 35-year-old man undergoes coverage of a soft-tissue defect on the posterior right elbow with a reverse lateral arm flap. He does not have a history of serious illness and has never smoked cigarettes. Vascular examination shows no abnormalities. Arterial blood to the flap is provided primarily by which of the following arteries?

A ) Artery to the biceps muscle

B ) Inferior cubital

C ) Musculocutaneous perforating branches from the brachioradialis muscle

D ) Posterior radial collateral

E ) Radial recurrent

A

The correct response is Option E.

The most likely dominant arterial supply to the reverse lateral arm flap is the radial recurrent artery. It is a branch of the radial artery and arises in the cubital fossa. It anastomoses with the posterior radial collateral artery just above the lateral epicondyle and medial to the brachioradialis within the lateral intermuscular septum. This pedicled flap can be used to cover defects of the elbow and requires retrograde flow through the posterior radial collateral artery via the radial recurrent artery. Prior trauma or radical surgery around the elbow and severe peripheral vascular disease warrant confirmation of presence or patency of the pedicle via Doppler ultrasonography or angiography. After rotating this flap more than 180 degrees, venous insufficiency may result and require microanastomosis of a superficial vein in the flap to a receptor vein in the defect to augment outflow.

The posterior radial collateral artery is the dominant inflow for the standard lateral arm flap. The inferior cubital artery is the dominant inflow and the musculocutaneous perforating branches from the brachioradialis muscle are the minor pedicles for the antecubital flap. The artery to the biceps muscle supplies the medial arm flap.

69
Q

A 60-year-old man undergoes debridement and coverage with a gracilis muscle free flap to correct exposed hardware 6 weeks after undergoing fracture fixation of the right ankle. During the hospital stay after the fracture fixation procedure, heparin was administered subcutaneously for deep venous thrombosis prophylaxis. The flap coverage procedure is complicated by thrombosis of the arterial anastomosis that requires thrombectomy and reanastomosis. Systemic heparin is administered because of the complication and continued postoperatively. Three days after this surgery, his right lower extremity, including the flap, is swollen and congested. Pulses in the leg are weak. Which of the following is the most appropriate test for this patient?

A ) Activated partial thromboplastin time (aPTT)

B ) D-dimer

C ) Factor V Leiden

D ) Platelet count

E ) Prothrombin time (PT)

A

E ) Prothrombin time (PT)

The correct response is Option D.

The most appropriate test is a platelet count to determine the possibility of heparin-induced thrombocytopenia (HIT) with thrombosis. This immune-mediated complication can occur in up to 3 to 5% of patients on heparin therapy, especially those previously exposed to heparin within the last 3 months of the second exposure. Approximately 20% of patients with HIT will have thrombotic events with potentially devastating consequences: 30% mortality and 30% limb loss. The pathogenesis of HIT involves the formation of multimolecular complexes between heparin and platelet factor 4. In some patients, immunoglobulin G-class antibodies are generated against the heparin: platelet factor 4 complexes. This results in potent platelet activation, platelet aggregation, and a marked increase in thrombin generation. The key to successful treatment is early recognition, and clinical diagnosis remains the €œgold standard. € A 30% decrease in baseline platelet count combined with any form of thrombosis in a patient receiving heparin should be considered heparin-induced thrombocytopenia and thrombosis until proven otherwise. The most essential element in the treatment of heparin-induced thrombocytopenia and heparin-induced thrombocytopenia and thrombosis remains discontinuation of all heparin, including heparin line flushes, subcutaneous heparin, and heparin-coated indwelling catheters. Starting alternative anticoagulant therapy (eg, danaparoid sodium, lepirudin, or argatroban) as soon as there is a strong clinical suspicion of HIT is advocated. This should be continued until platelet levels have returned to baseline. Although HIT is typically diagnosed on the basis of clinical signs, the diagnosis should be confirmed by laboratory testing whenever possible (platelet aggregation test

and/or specific heparin-platelet factor 4 enzyme-linked immunosorbent assay [ELISA] assay). In this patient, knowing if he may have HIT with thrombosis is essential because stopping the heparin drip becomes paramount.

D-dimer is a fibrin degradation product that is elevated in the presence of thrombosis. Factor V Leiden is the name given to a variant of factor V that causes hypercoagulability. Genetic tests are done to confirm this. PT is a measure of the extrinsic pathway of coagulation, and aPTT is a measure of both the intrinsic and common pathways of coagulation.

70
Q

A 7-year-old boy with sickle cell disease is brought to the emergency department after sustaining a Gustilo Type IIIB fracture of the lower extremity during an all-terrain vehicle collision. Soft-tissue coverage of exposed hardware and bone is planned. Which of the following factors is most likely to adversely affect perfusion to microsurgical reconstruction in this patient?

A ) High sympathetic tone

B ) Prostacyclines

C ) Sludging

D ) Young age of patient

A

The correct response is Option C.

Abnormally elevated rheologic factors associated with hematologic disorders such as sickle cell disease can seriously compromise perfusion. This usually happens in the form of sludging within the vessel lumen, causing flap compromise.

Microvascular surgery in young patients was once considered high risk because of misconceptions of higher sympathetic tone and higher propensity for spasm. This has been proven false. Many studies have proven microvascular surgery to be safe for children.

Humoral factors such as prostaglandin I2 (prostacyclin) and E1, as well as histamine and bradykinin, cause vasodilation.

Local factors such as acidosis, hyperthermia, and hypercapnia cause vasodilation.

71
Q

A 54-year-old man is scheduled for soft-tissue reconstruction of an esophageal defect caused by cancer. A free anterolateral thigh flap will be used. Harvest of the flap will most likely involve taking a cuff of which of the following muscles?

A ) Gracilis

B ) Rectus femoris

C ) Sartorius

D ) Tensor fascia lata

E ) Vastus lateralis

A

The correct response is Option E.

The anterolateral thigh flap is based on the descending branch of the lateral femoral circumflex vessels. Although classically thought of as providing septocutaneous perforators between the rectus femoris and the vastus lateralis muscle, increased familiarity with this flap and critical anatomical evaluation have shown that its perforators are primarily musculocutaneous through the vastus lateralis in the majority of cases. This anatomy is important to understand during flap elevation and harvest. Surgeons unfamiliar with these anatomical variations may find it confusing if no septocutaneous perforators are identified. They may unnecessarily abandon the anterolateral thigh flap because of this. However, once musculocutaneous perforators are found, it may be safer to harvest a cuff of vastus lateralis muscle with the perforators to maximize perfusion, as an intramuscular dissection can be tedious and risks damage to the perforators.

72
Q

A 45-year-old woman is scheduled to undergo delayed breast reconstruction using a transverse gracilis myocutaneous flap. Which of the following arteries provides the dominant blood supply of this flap?

A ) Deep femoral circumflex

B ) Lateral femoral circumflex

C ) Medial femoral circumflex

D ) Superficial femoral circumflex

A

The correct response is Option C.

The gracilis muscle arises from the anterior body and the inferior ramus of the pubis and the ischium. It passes distally in the medial thigh posterior to the long adductor and sartorius muscles and inserts on the medial aspect of the proximal tibia posterior, deep to the sartorius tendon and anterior to the semitendinous muscle insertion. Its innervation comes from a branch of the anterior division of the obturator nerve, which has 2 to 4 fascicles entering the muscle 6 to 10 cm from the origin.

The obturator nerve accompanies the dominant vascular pedicle, the medial femoral circumflex artery, and its venae comitantes arising from the profunda femoris artery and vein 8 to 12 cm from the muscle origin. A vascular pedicle can be obtained that is 4 to 6 cm long with a vessel diameter of 1 to 2 mm. Two minor vascular pedicles, which are branches of the superficial femoral artery, are located distally and may be sacrificed. No significant functional loss can be seen after removal of the gracilis muscle.

The transverse myocutaneous gracilis free flap with a transverse orientation of the skin paddle in the proximal third of the medial thigh region allows taking a moderate amount of tissue for autologous breast reconstruction in selected patients. The donor-site morbidity is similar to that of a classic medial thigh lift. The proximal pedicle enters the gracilis muscle 8 to 12 cm below the pubic tubercle and sends both septocutaneous and musculocutaneous perforators. These branches have a pronounced tendency to travel in a transverse direction, supplying the cutaneous territory over the long adductor and sartorius anteriorly and extending for greater than 5 cm beyond the posterior margin of the gracilis muscle. This flap is reliable and consistent.

73
Q

A 52-year-old woman undergoes reconstruction of the left breast using an ipsilateral extended latissimus dorsi myocutaneous pedicled flap. Postoperatively she develops venous congestion along the distal end of the skin paddle. Leeches are used in an attempt to relieve venous congestion. Which of the following is the most effective prophylactic antibiotic therapy?

A ) Cephalexin

B ) Ciprofloxacin

C ) Clindamycin

D ) Metronidazole

E ) Penicillin

A

The correct response is Option B.

Medicinal leeches (Hirudo medicinalis) have been used as an aid to salvage congested free flaps. The incidence of infection associated with leech therapy reported in the literature ranges from 2.4% to 20%. The severity of infections ranges from cellulitis to tissue necrosis, abscess, and septicemia. Delayed infection from colonized necrotic tissue can also occur. The organism most often responsible is Aeromonas hydrophila, a gram-negative anaerobe that is part of the intestinal resident flora of the leech. Resistance to penicillins and first-generation cephalosporins is not uncommon because of their production of beta-lactamase enzymes. Resistance to trimethoprim and sulfamethoxazole, tetracycline, imipenem, and even gentamicin has also been reported. Susceptibility to fluoroquinolones such as ciprofloxacin continues to be observed. Prophylactic antibiotics with a fluoroquinolone and aminoglycoside are recommended, and therapy should be continued until any open wound or necrotic tissue has completely healed.

74
Q

A 67-year-old man undergoes three-vessel coronary artery bypass grafting using the left and right internal mammary vessels and a saphenous vein graft. He develops mediastinitis postoperatively and requires wide operative debridement with a resultant chest defect. Reconstruction with which of the following flaps is most likely to result in flap necrosis?

A ) Latissimus dorsi

B ) Omentum

C ) Pectoralis major

D ) Rectus abdominis

E ) Serratus anterior

A

The correct response is Option D.

The rectus abdominis muscle has several sources of blood supply, including the superior and inferior epigastrics and intercostal vessels. For the purpose of chest wall reconstruction, the rectus muscle is pedicled upon its superior blood supply, the superior epigastric vessels, which itself is a continuation of the internal mammary vessels. Since the internal mammaries have been used on both sides for the purpose of coronary artery bypass grafting, the blood supply to the rectus has been compromised. Although it is possible to raise the rectus muscle on only the eighth intercostal blood vessel, this method is less reliable and less likely to be sufficient to sustain the entire rectus muscle, making it prone to flap necrosis.

75
Q

A 35-year-old man is undergoing repair of a pressure sore on the left ischium using the musculocutaneous flap shown in the photograph. Which of the following is the Mathes and Nahai classification of this flap?

A ) Type I

B ) Type II

C ) Type III

D ) Type IV

E ) Type V

A

The correct response is Option C.

The gluteal musculocutaneous flap is a Type III flap, meaning it has two dominant pedicles (the superior and inferior gluteal arteries). These arteries are separated by the piriformis muscle and are sourced to the internal iliac system.

A Type I flap has a single dominant pedicle (ie, tensor fascia lata). A Type II flap has dominant and minor pedicle(s) (ie, gracilis). A Type IV flap has multiple segmental pedicles (ie, sartorius). A Type V flap has a dominant pedicle and secondary segmental pedicles (ie, latissimus).

76
Q

When used for breast reconstruction, both the superior and the inferior gluteal artery perforator flaps utilize a vascular bundle that is a terminal branch of which of the following arteries?

A ) Deep circumflex iliac

B ) External iliac

C ) Femoral

D ) Internal iliac

E ) Pudendal

A

The correct response is Option D.

Both the superior and inferior gluteal arteries are terminal branches of the internal iliac artery. They pass out of the pelvis above and below the piriformis muscle, supplying the upper and lower halves of the muscle, respectively. As the superior gluteal artery passes the greater sciatic foramen, it divides into superficial and deep branches. The deep branch travels between the gluteus medius muscle and the iliac bone. The superficial branch goes on to supply the gluteus muscle and the overlying skin territory. The superficial branch of the gluteal artery nourishes the fat and skin in the musculocutaneous flaps in this region. These perforating vessels can be separated from the underlying muscle and fascia and form the basis for the S-GAP flap, which allows maximal preservation of the donor site muscle and other underlying structures while creating a reliable skin €“soft-tissue flap. Two to three perforators are usually found arising from this vessel, with a pedicle length between 3 cm and 8 cm.

77
Q

A 35-year-old man sustains a traumatic injury to the lower leg and undergoes closure of the wound using an anterolateral thigh free flap (shown). A cuff of muscle is harvested with the flap to fill a bone defect. Which of the following muscles can be safely harvested while using the same vascular pedicle as the flap?

A ) Adductor longus

B ) Rectus abdominis

C ) Vastus intermedius

D ) Vastus lateralis

E ) Vastus medialis

A

The correct response is Option D.

The anterolateral thigh flap is located over the lateral third of the thigh, between the borders of the rectus femoris and vastus lateralis muscle. Its blood supply comes from perforating branches of the lateral circumflex femoral artery and its venae comitantes. These vessels arise from the profunda femoris artery and vein. By utilizing the transverse branch of the lateral femoral circumflex artery and venae comitantes and their musculocutaneous perforators, the vastus lateralis muscle can be harvested with the anterolateral thigh flap. This muscle can be used for added bulk in the flap. The anterolateral thigh flap may also be harvested with the tensor fascia lata muscle. This flap may also be used as a functional muscle transfer.

78
Q

A 27-year-old man has a large segmental defect of the humerus. Reconstruction using an osteocutaneous free flap from the lower leg is planned. A portion of which of the following muscles is most appropriately included with the bone to protect the pedicle and improve reliability of the skin island?

A ) Extensor digitorum longus

B ) Flexor hallucis longus

C ) Gastrocnemius

D ) Peroneus longus

E ) Tibialis posterior

A

The correct response is Option B.

The most appropriate muscle is the flexor hallucis longus.

The fibular free flap is a workhorse flap for reconstruction of large segmental bone defects. The vascular supply of this flap comes from the peroneal artery. A large segment of bone can be harvested, consisting of the majority of the fibula with the exception of the proximal 6 cm at the fibular head and the distal 6 cm near the ankle joint. The fibular flap can be harvested with a skin paddle on the lateral aspect of the leg, based on perforators through the lateral intermuscular septum or via the muscle. The pedicle is located adjacent to the flexor hallucis longus muscle in the deep posterior compartment of the leg. Inclusion of a cuff of the flexor hallucis longus muscle can be performed to protect the pedicle and add bulk to the reconstruction if necessary. A composite flap of fibula and the entire flexor hallucis longus muscle has been described. The lateral portion of soleus muscle can also be included in the fibula free flap if desired.

The extensor digitorum longus muscle is in the anterior compartment of the leg. The gastrocnemius muscle is in the posterior compartment of the leg and is more superficial. The peroneus longus muscle is in the lateral compartment of the leg. The tibialis posterior muscle is in the deep posterior compartment of the leg and is located along the interosseous membrane.

79
Q

A 55-year-old man has complete excision of squamous cell carcinoma of the scalp resulting in a 6 x 8-cm occipital defect with exposed calvaria. Reconstruction with a lower third trapezius island flap is performed. Which of the following arteries is the major blood supply for the flap?

(A) Dorsal scapular

(B) Occipital

(C) Superficial cervical

(D) Thoracoacromial

(E) Thoracodorsal

A

The correct response is Option A.

The predominant blood supply of the inferior portion of the trapezius myocutaneous flap is the dorsal scapular artery. The dorsal scapular artery is a separate branch of the subclavian artery or forms a common trunk with the superficial cervical artery. When a common trunk is present, it is referred to as the transverse cervical artery, the superficial cervical artery as the superficial branch of the transverse cervical artery, and the dorsal scapular artery as the deep branch of the transverse cervical artery.

The superior or descending part of the trapezius is supplied by branches of the occipital artery. The middle or transverse part is supplied by the superficial cervical artery. The thoracoacromial artery supplies the pectoralis major muscle flap and the thoracodorsal artery supplies the latissimus dorsi muscle flap.

80
Q

A patient with a complex defect that requires replacement of skin, muscle, and bone is scheduled to have reconstruction with a single free flap from the subscapular system. Which of the following would NOT be used as part of this reconstruction?

(A) Parascapular fasciocutaneous tissue

(B) Serratus anterior muscle

(C) Trapezius muscle

(D) Vascularized rib

(E) Vascularized scapular bone

A

The correct response is Option C.

Unlike the other tissues, the trapezius muscle is based on the transverse cervical artery (based on the thyrocervical trunk in 80% of cases or the subclavian artery in 20% of cases). The subscapular system allows for the creation of chimeric flaps that can include bone, muscle, fascia, fat, and skin. Flaps that have been used include the serratus anterior muscle and fascia, latissimus dorsi muscle and fascia, scapular and parascapular fascia and overlying skin, and scapular and rib bone. Such combinations allow for versatility in reconstructing complex three €‘dimensional defects that are often encountered in the face.

81
Q

Which of the following arteries is the major blood supply to the flap most often used in the reconstruction of open-knee defects?

(A) Anterior tibial

(B) Medial circumflex femoral

(C) Peroneal

(D) Posterior tibial

(E) Sural

A

The correct response is Option E.

The medial and lateral sural arteries supply circulation to the gastrocnemius muscle, which, when released and rotated proximally, provides the best coverage for complex knee wounds. The gastrocnemius muscle may be harvested with overlying skin or may be skin-grafted.

The medial circumflex femoral artery and profunda femoris vessels supply the gracilis muscle, which provides coverage for medial thigh and groin defects. It cannot be rotated distally.

The posterior tibial artery provides some blood supply to the soleus muscle, which is the preferred muscle coverage to wounds to the middle one third of the lower extremity. The posterior tibial, peroneal, and anterior tibial arteries provide perforators for fasciocutaneous flaps in the lower extremity, which are limited in their applicability and are too distal for coverage of the wound.

82
Q

An 84-year-old man comes to the office because he has had pain and purulent drainage from the left groin incision site since undergoing a left common iliac artery to common femoral bypass procedure performed for critical stenosis of the iliac artery three weeks ago. Temperature is 38.8 °C (101.8 °F), blood pressure is 140/90 mmHg, and pulse rate is 100/min. Physical examination shows an exposed vein graft at the groin incision site. A magnetic resonance angiogram shows a patent bypass graft; the superficial femoral artery is patent, but a profundus branch is occluded. Following debridement, wound coverage with which of the following flaps is the most appropriate next step?

(A) Anterior lateral thigh

(B) Gracilis muscle

(C) Rectus femoris muscle

(D) Sartorius muscle

(E) Vastus lateralis muscle

A

The correct response is Option D.

The sartorius muscle would provide the most reliable option for wound coverage for the patient discussed. The sartorius muscle flap originates from the anterior iliac spine and inserts into the medial tibia. Eight to eleven perforators off the superficial femoral artery segmentally supply the sartorius, a type V muscle flap. The proximal pedicle is located approximately 6 cm from the anterior superior iliac spine. The sartorius muscle is adjacent to the femoral vessels.

The anterior lateral thigh, rectus femoris, and vastus lateralis muscles all are supplied by branches of the lateral femoral circumflex vessel. The gracilis muscle is supplied by the medial femoral circumflex artery. Both medial and lateral circumflex arterial branches originate from the proximal portion of the profunda femoris artery. Occlusion of the profunda would make the use of these flaps risky and may result in flap loss.

The rectus femoris muscle is a type I muscle flap with a single dominant blood supply from the lateral femoral circumflex artery.

83
Q

A 59-year-old man comes to the office for follow-up examination three weeks after undergoing left total knee replacement. Physical examination shows a draining sinus on the distal aspect of the knee. Surgical removal of the hardware and flap reconstruction are planned. Which of the following is the dominant pedicle to the muscle flap that is most appropriate for reconstruction?

(A) Anterior tibial artery

(B) Ascending branch of the medial circumflex femoral artery

(C) Medial sural artery

(D) Perforating branch of the distal superficial femoral artery

(E) Proximal branches of the posterior tibial artery

A

The correct response is Option C.

The muscle flap best suited to reconstruct this defect is a medial head of gastrocnemius muscle rotation flap. Its dominant pedicle is the medial sural artery, which is a branch of the popliteal artery.

The ascending branch of the medial circumflex artery is the dominant pedicle to the gracilis flap, whereas the minor pedicles are perforating branches of the distal superficial femoral artery. A proximally based gracilis flap would not have the arc of rotation to adequately cover the knee. A distally based flap would require a delay procedure and is not the best choice for this defect.

The lateral sural artery is the dominant blood supply to the lateral head of the gastrocnemius, which is smaller, and, therefore, not as well suited for coverage of large patellar defects such as this one.

The soleus is based off the proximal two branches of the posterior tibial artery, branches of the proximal popliteal, and branches of the peroneal artery. It is used for reconstruction of defects of the middle third of the lower extremity and does not have an arc of rotation sufficient enough to cover this defect.

84
Q

The dominant pedicle of the gracilis muscle flap is located between the adductor longus and which of the following muscles?

(A) Adductor brevis

(B) Adductor magnus

(C) Pectineus

(D) Sartorius

(E) Semitendinosus

A

The correct response is Option A.

The dominant pedicle of the gracilis muscle flap is located between the adductor longus and adductor brevis muscles. The pectineus, adductor magnus, sartorius, and semitendinosus muscles are anatomically incorrect relative to the location of this pedicle.

Although the origin of the dominant vascular pedicle to the gracilis muscle flap may originate from either the profundus femoris or medial circumflex femoral vessels, the course of this vascular pedicle is consistent and runs between the adductor longus and adductor brevis muscles. This vascular pedicle enters the gracilis muscle approximately 8 to 10 cm below the pubic tubercle. The superficial position of the gracilis muscle, its thin, ribbon €‘like character, and also its consistent vascular anatomy allow this muscle flap to remain one of the most useful muscles in the field of microvascular surgery. Although the dominant vascular pedicle easily nourishes the entire muscle, the distal third of the skin overlying the gracilis muscle does not receive sufficient blood supply for reliable transfer.

85
Q

During dissection for a microsurgical free parascapular flap, the circumflex scapular artery is located in the triangular space bordered by the teres minor, the long head of the triceps, and which of the following muscles?

(A) Infraspinatus

(B) Latissimus dorsi

(C) Subscapularis

(D) Teres major

(E) Trapezius

A

The correct response is Option D.

The scapular area supplied by the circumflex scapular artery is a commonly used donor site in reconstructive microsurgery because of the consistent and reliable vascular anatomy, easily accessible pedicle, and good vessel diameter and length. The circumflex scapular artery arises from scapular artery approximately 3 to 4 cm from its origin at the axillary artery. It then passes through the triangular space, which is bordered above by the teres minor, the teres major below, and the long head of triceps laterally. Flap size can reach 10 × 25 cm with generally closure of the defect directly with acceptable scarring. Parascapular osteocutaneous flaps can also be harvested using the lateral border of the scapula, which is useful for reconstruction of the hand and face.

86
Q

A 28-year-old man is brought to the emergency department 30 minutes after he sustained avulsion injuries to the nondominant left hand when it became caught in a motor vehicle fanbelt. Physical examination shows amputation of the index finger at the level of the proximal interphalangeal joint as well as a 2 x 1-cm area of soft-tissue loss. Replantation of the amputated digit is performed, and the resulting 2 x 1-cm soft-tissue avulsion volar defect is covered with an arterialized venous flow-through flap with overlying skin interposed as a vein graft in the arterial repair. Which of the following is the most likely early complication of this flap procedure?

(A) Arterial thrombosis

(B) Congestion of the flap

(C) Failure of the replantation

(D) Hematoma from vessel leak

(E) Loss of flap due to infection

A

The correct response is Option B.

Venous flow €‘through flaps (VFTFs) are unusual but are gaining acceptance for certain kinds of hand and finger wounds. The ideal site for coverage with a VFTF is a long and narrow defect needing thin soft tissue. VFTFs typically become congested in the first week and then decongest over the following two weeks as they revascularize from the wound bed. VFTFs cannot reliably transfer composite tissue such as bone and tendon or cover a wide defect such as an entire palm. Because VFTFs do not bring in vascularization to the wound bed as well as classic flaps, they are not indicated in radiated or potentially infected wound beds. A small defect such as the 2-cm defect needing coverage during the replantation of the finger in the scenario described is the ideal candidate for this flap.

When compared with simple vein grafts, VFTFs are not associated with increased rates of arterial thrombosis, failure of replantation, or hematoma. Although VFTFs are more susceptible to infection than typical flaps, congestion of the flap with superior epidermolysis is a much more likely complication.

87
Q

A 22-year-old man has undergone multiple debridements of a wound of the right hand since he sustained a gunshot wound six days ago. Physical examination shows a significant soft-tissue defect and a 4-cm segment of bone loss from the third metacarpal. A radial forearm osteofasciocutaneous flap procedure for simultaneous reconstruction of the soft-tissue and skeletal defects is planned. Which of the following portions of the radius is the most appropriate site for harvesting the cortical bone graft?

(A) Radial aspect, between the brachioradialis and pronator teres insertions

(B) Radial aspect, between the brachioradialis insertion and flexor pollicis longus origin

(C) Radial aspect, between the pronator teres insertion and flexor pollicis longus origin

(D) Ulnar aspect, between the brachioradialis and pronator teres insertions

(E) Ulnar aspect, between the pronator teres insertion and flexor pollicis longus origin

A

The correct response is Option A.

When raising a radial forearm osteofasciocutaneous flap, cortical bone should be harvested from the radial aspect of the radius between the brachioradialis and pronator teres muscle insertions. A cuff of the flexor pollicis longus to the radius is maintained to preserve the periosteal vessels. A small cuff of periosteum is maintained on both sides of the radial vessels. All of the other optional donor sites are anatomically not appropriate. A segment up to 10 cm in length and up to 40% of the cross €‘sectional area of the radius may be harvested. The superficial branch of the radial nerve, running adjacent to the brachioradialis, is at risk and is sometimes sacrificed.

88
Q

Which of the following commonly used muscle flaps provides the versatility of two dominant vascular supplies?
(A) Gastrocnemius
(B) Latissimus dorsi
(C) Pectoralis major
(D) Rectus abdominis
(E) Sartorius

A

The correct response is Option D.

The Mathes and Nahai classification of muscle flaps is based on vascular supply. Type III muscles have two dominant pedicles. The rectus abdominis muscle is supplied by both the superior and inferior deep epigastric vessels. The superior epigastric pedicle is an extension of the internal mammary circulation and allows transposition of the rectus muscle to the chest for breast and thoracic reconstruction. Pedicle flaps based on the inferior circulation are useful for pelvic and abdominal reconstruction. If one end of the muscle is in the zone of injury and the other side is spared, then a portion of the muscle could still be available for flap transfer.

The gastrocnemius muscle flap is an example of a type I flap, having a single dominant circulation. Injury to the upper posterior calf would preclude safe use as a flap.

Both the latissimus and pectoralis major muscles are type V, containing both a dominant pedicle as well as a group of smaller, equally sized segmental vessels. These muscles may be elevated as flaps based on either the dominant circulation or on the group of segmental vessels, and therefore, also provide a degree of versatility.

The sartorius muscle is classified as type IV, having only a segmental circulation of multiple pedicles of equal size rather a dominant pedicle. The sartorius muscle is useful for coverage of the femoral sheath after a vascular surgical wound dehiscence with exposed prosthetic graft.

Type II muscle flaps have both a dominant pedicle and a minor circulation. The minor vessels are typically divided to allow use of a flap based on the dominant pedicle. Examples include the gracilis and biceps femoris muscles.

89
Q

A 54-year-old man has a nonhealing wound of the lateral aspect of the right ankle (shown) three months after he underwent open reduction and internal fixation of a fracture. Physical examination shows exposed bone at the base of the wound but no evidence of deep infection. Which of the following flap procedures is most appropriate for closure of this patient=s defect?
(A) Bilateral V-to-Y advancement flaps
(B) Dorsalis pedis fasciocutaneous flap
(C) Extensor digitorum brevis muscle flap
(D) Flexor digiti minimi muscle flap
(E) Lateral calcaneal flap

A

The correct response is option E.

The most appropriate procedure to close this defect is a lateral calcaneal flap, which has an axial pattern based on the lateral calcaneal artery, a terminal branch of the peroneal artery. This flap is very reliable, even in older patients, and lateral calcaneal artery patency and flow can be determined by preoperative Doppler examination. The donor site is usually skin grafted. (See photographs below.) Other acceptable options include reverse sural flap and free tissue transfer.

Bilateral V-to-Y advancement would be difficult in this patient because there is edema and inflammation associated with a chronic wound. The dorsalis pedis fasciocutaneous flap would reach this defect but has significant donor site morbidity and should be reserved for situations in which other options are not available. The flexor digiti minimi muscle flap is lateral on the foot but is not able to reach the lateral ankle. The extensor digitorum brevis muscle flap has a large arc of rotation but requires sacrifice of the dorsalis pedis artery.

90
Q

A 39-year-old woman with exposed hardware after spinal fusion undergoes an external oblique turnover muscle flap procedure. The blood supply to this flap is primarily from which of the following vessels?
(A) Deep circumflex iliac
(B) Deep inferior epigastric
(C) Iliolumbar
(D) Intercostal
(E) Subscapular

A

The correct response is Option D.

The external oblique turnover muscle flap is not commonly used but has the potential to provide adequate coverage of large defects of the back that extend to the midline at the level of T10 to L4. The upper half of the flap is supplied by the 4th through 11th intercostal arteries, whereas the lower half is supplied by one or two vessels from the deep circumflex iliac artery (95%) or the iliolumbar artery (5%). The upper half has a strictly segmental blood and nerve supply, whereas the lower half has segmental innervation but derives its blood supply from one artery.

91
Q

The direct vascular supply to the arm flap shown is which of the following arteries?
(A) Posterior interosseous
(B) Posterior radial collateral
(C) Profunda brachial
(D) Recurrent radial
(E) Superior ulnar collateral

A

The correct response is Option B

Flaps require preservation of the defined arterial and venous supply. Regional flaps can be based distally or proximally. The lateral arm flap was initially described by Song and popularized by Matloub, et al. and Katsaros, et al. This flap has considerable application in reconstruction of the upper extremity. It can be harvested from the same extremity; it has thin, pliable skin; and it can be innervated. The lateral arm flap can also include a segment of vascularized humerus or a segment of vascularized triceps tendon. The flap can also be segmentally contoured for hand defects. The lateral arm free flap is based on the posterior radial collateral artery, which is a branch of the profunda brachial artery. A reversed pedicled lateral arm flap can also be performed. This reverse flap is based on the radial recurrent artery. The superior ulnar collateral artery is the dominant pedicle of the medial arm flap. The medial arm flap is also useful for upper extremity coverage, but there are significant variations in the superior ulnar collateral artery which makes this flap a less-than-optimal choice for microvascular surgical reconstruction. The posterior interosseous artery is the dominant blood supply of the posterior interosseous artery flap. The posterior interosseous artery flap is also used for reconstruction of the hand because it has thin, pliable skin. As a pedicle flap, it can cover the wrist and extend to the first web space. Use of the posterior interosseous flap can compromise motor nerves to the extensor carpi ulnaris or extensor digiti quinti.

92
Q

Which of the following muscles is included in a facial artery musculomucosal flap?

(A) Buccinator
(B) Depressor anguli oris
(C) Levator labii superioris
(D) Orbicularis oris
(E) Zygomaticus major

A

The correct response is Option A.

Because the buccinator muscle is sandwiched between the facial artery and the oral mucosa, it must be included in a facial artery musculomucosal flap. The buccinator muscle originates from the pterygomandibular raphe and inserts into the orbicularis oris muscle and mucosa of the lateral lip elements.
The other muscles listed are supplied by the facial nerve but are not included in a facial artery musculomucosal flap because of their location. The depressor anguli oris and zygomaticus major muscles lie superficial to the plane of dissection for this flap. The levator labii superioris muscle inserts into the lateral half of the lip, but its origin is from the infraorbital margin of the maxilla. The orbicularis oris muscle is anterior to the plane of dissection for the facial artery musculomucosal flap.

93
Q

A 65-year-old man undergoes operative removal of a basal cell carcinoma at the junction of the upper cheek and temporal region, followed by coverage of the resultant 10 _ 5-cm defect with a submental myocutaneous flap. This flap derives its blood supply from a branch of which of the following arteries?

(A) Facial
(B) Inferior thyroid
(C) Lingual
(D) Superior thyroid
(E) Transverse cervical

A

The correct response is Option A.

The submental flap is a myocutaneous flap that is useful in head and neck reconstruction. This flap provides a contour, color, and tissue texture that is suitably matched to the face. The flap is elevated below the level of the platysma muscle and includes the submental artery and vein, which are direct branches of the facial artery and vein. The flap can be transposed to cover defects in the lower and central thirds of the face and into the inferior aspect of the upper third of the face.
The submental artery is a consistent branch of the facial artery and gives off one or two cutaneous perforators to the submental skin. The submental artery runs in relation to the anterior belly of the diagastric muscle. Of the choices listed, the facial artery is the most superior branch of the external carotid artery. The lingual artery provides the blood supply to the tongue, the superior thyroid and inferior thyroid arteries provide the blood supply to the thyroid gland, and the transverse cervical artery gives off a descending branch, which provides the blood supply to the trapezius muscle.

94
Q

Which of the following arteries is the basis of the major blood supply to the pectoralis major myocutaneous flap for head and neck reconstruction?

(A) Internal mammary
(B) Lateral thoracic
(C) Superior thoracic
(D) Thoracoacromial
(E) Transverse cervical

A

The correct response is Option D.

The major blood supply to the pectoralis major myocutaneous flap is the thoracoacromial artery. This flap has been the workhorse for head and neck reconstruction. It remains a lifeboat flap for microvascular flap failures in the area. It arises from the second part of the axillary artery (continuation of the subclavian artery) and divides into four branches with the pectoral branch supplying the pectoral muscles and the flap. The superior thoracic artery arises from the first part of the axillary artery and also supplies some part of the pectoral muscles. The lateral thoracic artery also arises from the axillary artery but does not supply the flap. The internal mammary artery does supply the pectoralis major muscle and its accompanying skin; however, it cannot be pedicled on this axis for head and neck reconstruction. The transverse cervical artery originates from the subclavian artery and supplies the muscles of the neck and scapula.

95
Q

Which of the following interventions is most appropriate to improve the viability of an ischemic skin flap?

(A) Apply medicinal leeches to the flap
(B) Apply nitroglycerin paste to the flap
(C) Elevate the affected area
(D) Ensure adequate fluid resuscitation
(E) Perform hyperbaric oxygen therapy

A

The correct response is Option D.

The initial 24-hour period is critical to flap viability. In cases of compromised arterial inflow, steps that can be taken to improve arterial inflow are of primary concern. Ensuring adequate postoperative fluid resuscitation is paramount. This ensures adequate cardiac output and optimizes tissue perfusion.

Medicinal leeches are indicated for venous congestion and would have questionable value in an ischemic setting. Although hyperbaric oxygen therapy and elevation may both have beneficial effects in this setting, they are less appropriate than fluid resuscitation. Some clinicians will use 2% nitroglycerin ointment to ischemic areas every four to six hours or silver sulfadiazine cream twice daily. Either of these techniques is believed to be useful in reducing the risk and extent of full-thickness skin loss but would probably not be as important as ensuring adequate hydration.

96
Q

What is the theoretic gain in length achieved by performing a Z-plasty with angles of 75 degrees?

(A) 25%
(B) 50%
(C) 75%
(D) 100%

A

The correct response is Option D.

The Z-plasty is a technique in which pairs of triangular transposition flaps are created adjacent to a scar and then transposed across the scar, increasing the length of the central limb and changing the orientation of the scar. This technique can be used in patients undergoing burn reconstruction to lengthen linear scar contractures, disperse linear scars, and realign scars within the lines of minimal tension. The theoretic gain in length correlates directly with the angle and length of the flap limbs. The actual gain in the length of the central axis will be decreased by 30% to 50% because of the contractile properties of skin.

The theoretic gain achieved with each angle is illustrated in the table below.

A 60-degree Z-plasty is performed most commonly because it produces a significant gain in length while minimizing the tension of closure.

97
Q

According to the Mathes/Nahai classification of muscle and musculocutaneous flaps, which of the following is a type III flap?

(A) Gastrocnemius
(B) Gluteus maximus
(C) Latissimus dorsi
(D) Pectoralis major
(E) Vastus lateralis

A

The correct response is Option B.

The gluteus maximus and rectus abdominis muscle flaps have a type III vascular pattern consisting of dual dominant pedicles.

Type I muscle flaps have one dominant pedicle; examples include the gastrocnemius, tensor fascia lata, and vastus lateralis flaps. In type II muscle flaps, such as the gracilis and trapezius, there is one dominant pedicle and multiple secondary pedicles. The external oblique and sartorius muscle flaps have a type IV vascular pattern, which is characterized by a segmented blood supply. Type V muscle flaps, such as the latissimus dorsi and pectoralis major, have one dominant pedicle and multiple secondary segmental pedicles.

98
Q

A 20-year-old woman is undergoing evaluation eight months after sustaining a severe degloving injury, including multiple injuries to skin and tendons, to the dorsal aspect of the left hand. At the time of injury, extensive wound care was followed by full-thickness skin grafting from the left side of the groin. On current physical examination, the patient has severe contractures and clawing of the metacarpophalangeal joints. The photograph above shows an outline of the flap that is to be used to resurface the dorsal aspect of the hand.

This flap derives its axial blood supply from which of the following arteries?

(A) Deep inferior epigastric artery
(B) Medial femoral circumflex artery
(C) Superficial circumflex iliac artery
(D) Superficial external pudendal artery
(E) Superficial inferior epigastric artery

A

The correct response is Option E.

The flap depicted in the photograph is the superficial inferior epigastric artery (SIEA) flap, also known as the Shaw flap. This flap derives its axial blood supply from the superficial inferior epigastric (SIE) artery, one of three cutaneous arteries supplied by the common femoral artery. The SIE artery arises from the intersection of the inguinal ligament and the femoral artery, then courses superiorly and laterally toward the anterior axillary line.

The SIEA flap is typically transferred as a pedicled flap, but fasciocutaneous free tissue transfer has also been described. This flap is recommended for coverage of wounds of the hand and forearm because it lies in a relatively higher position on the torso than the groin flap; this allows for improved mobility of the shoulder while placing the elbow in a comfortable, flexed position while the flap matures. To confirm the pedicle pattern, Doppler ultrasonography can be performed prior to elevation of the flap. A photograph of the inset of the flap is shown above.

The superficial circumflex iliac artery provides the axial blood supply to the groin flap. This artery also originates from the femoral artery and travels parallel to the inguinal ligament, approximately 1 cm deep to the ligament.

Although the deep inferior epigastric artery supplies the rectus muscle on its deep surface, it does not supply the skin of the abdomen directly; instead, perforators emerge from the deep inferior epigastric artery and course through the rectus muscle to supply blood to the skin of the abdomen. This vascularity allows the skin and muscle to be harvested for free flap transfer.

The superficial ex

99
Q

Immediately after undergoing reconstruction of a wound of the dorsal aspect of the hand using a reverse radial forearm flap, a patient has marked venous congestion of the flap. There is no hematoma visible under the flap. Which of the following is the most appropriate management?

(A) Increasing the temperature of the hand
(B) Elevation of the hand
(C) Intravenous infusion of heparin
(D) Application of leeches to the flap
(E) Anastomosis of an outflow vein

A

The correct response is Option E.

When harvesting a reverse island flap such as the reverse radial forearm flap, the surgeon should always attempt to preserve an outflow vein, which will be necessary for drainage of the flap if the reverse flow venous system does not function adequately. Most patients exhibit only mild venous congestion, and leeches can be applied to drain the flap sufficiently and thus preserve it. However, any patient who has immediate onset of marked venous congestion in which the cause is not obvious (ie, kinking of the pedicle or hematoma under the flap) should undergo immediate anastomosis of an outflow vein to decompress the flap.

Conservative measures such as increasing the temperature of the hand, elevating the extremity, or infusing heparin intravenously will ultimately fail in a patient with marked venous congestion requiring immediate operative treatment.

100
Q

The deep inferior epigastric artery arises from which of the following arteries?

(A) External iliac
(B) Femoral
(C) Internal iliac
(D) Internal mammary
(E) Superficial inferior epigastric

A

The correct response is Option A.

The deep inferior epigastric artery and vein arise from the external iliac artery at a point just proximal to where the artery passes beneath the inguinal ligament.

The internal mammary vessels provide the regional source for the superior epigastric artery. The femoral, internal iliac, and superficial inferior epigastric arteries are not source vessels for the deep inferior epigastric artery and vein. The femoral artery originates as the external iliac vessels exit from the inguinal ligament. The internal iliac artery supplies blood to structures within the pelvis.

101
Q

The third perforating branch of the profunda femoris artery, which is the blood supply for the lateral thigh flap, originates at a level that is immediately caudad to which of the following muscles?

(A) Adductor brevis
(B) Adductor longus
(C) Adductor magnus
(D) Gracilis
(E) Pectineus

A

The correct response is Option A.

The third perforating branch of the profunda femoris artery provides the predominant blood supply for the lateral thigh flap. The third perforating branch originates immediately caudad to the adductor brevis muscle, pierces the insertion of the adductor magnus, courses superficially, and then traverses between the biceps femoris and the vastus lateralis. It pierces the deep fascia at this point, and then emerges at the midpoint between the greater trochanter and lateral epicondyle of the femur.

The lateral thigh flap is a versatile flap that has a thin, cutaneous skin paddle and is innervated by the lateral cutaneous nerve of the thigh. As much as 8 * 25 cm can be transferred; however, split-thickness skin grafting is required for closure at the donor site if a flap with a width of greater than 8 cm is harvested. In addition, the lateral aspect of the thigh flap can be hair-bearing, especially in men, while the dermis is characteristically thick and stiff.

The adductor longus muscle originates at the pubic ramus and inserts on the medial epicondyle. The gracilis muscle in the thigh is perfused by the medial circumflex artery, while the pectineus muscle receives its vascularity from the first perforating branch of the profunda femoris.

102
Q

According to the Mathes-Nahai classification of muscle and musculocutaneous flaps, which of the following is a type I flap?

(A) Gluteus maximus
(B) Gracilis
(C) Latissimus dorsi
(D) Sartorius
(E) Tensor fascia lata

A

The correct response is Option E.

The tensor fascia lata, gastrocnemius, and vastus lateralis muscle flaps have a type I vascular pattern consisting of one dominant pedicle. These muscles can be used as rotation flaps for regional reconstruction or as free tissue for transfer.

Type II muscle flaps have one dominant pedicle and several minor pedicles; examples include the abductor digiti minimi, gracilis, and soleus. In type III muscle flaps, such as the gluteus maximus, rectus abdominis, and serratus, there are two dominant vascular pedicles. The extensor hallucis longus, sartorius, and tibialis anterior muscle flaps have a type IV vascular pattern, with multiple segmental vascular pedicles entering along the course of the muscle belly. Type V muscle flaps, such as the latissimus dorsi and pectoralis major, have one dominant pedicle and secondary segmental pedicles, which provide a significant source of muscle circulation.

103
Q

Which of the following muscles comprise the boundaries of the triangular space in which the circumflex scapular artery is located?

(A) Infraspinatus, latissimus, and teres minor
(B) Latissimus, long head of the triceps, and teres major
(C) Latissimus, teres minor, and teres major
(D) Long head of the triceps, supraspinatus, and teres major
(E) Long head of the triceps, teres major, and teres minor

A

The correct response is Option E.

The boundaries of the triangular (or omotricipetal) space, in which the circumflex scapular artery is located, are comprised of the long head of the triceps muscle laterally, the teres major muscle inferiorly, and the teres minor muscle superiorly. The circumflex scapular artery is a branch of the subscapular artery and emerges from the triangular space to supply blood to the parascapular flap.

The quadrangular space can be found immediately lateral to the triangular space. This space is defined by the surgical neck of the humerus, lateral head of the triceps muscle, teres major muscle, and teres minor muscle. The axillary nerve and posterior humeral circumflex artery pass through this space.

The other muscles listed above can be identified in the region of the shoulder and trunk but do not define the triangular space.

104
Q

Which of the following muscles comprise the borders of the triangular space?

(A) Anterior border of the scapula, serratus anterior, and long head of the triceps muscles
(B) Latissimus dorsi, teres major, and teres minor muscles
(C) Medial border of the scapula, rhomboideus major, and rhomboideus minor muscles
(D) Triceps, teres major, and teres minor muscles

A

The correct response is Option D.

The triceps, teres major, and teres minor muscles comprise the borders of the triangular space. A knowledge of this region is essential to harvest the scapular and parascapular flaps, which receive their blood supply from the circumflex scapular artery. The pedicle of the parascapular flap arises from the triangular space and enters the deep surface of the flap.

The other muscles listed above can be identified in the region of the shoulder and trunk but do not define the triangular space.

105
Q

In a patient who is undergoing dissection of a gracilis musculocutaneous flap, the gracilis muscle can be identified immediately posterior to which of the following muscles in the thigh?

(A) Adductor longus muscle
(B) Adductor magnus muscle
(C) Pectineus muscle
(D) Sartorius muscle

A

The correct response is Option A.

In order to effectively identify the gracilis muscle prior to flap harvest, the patient should be placed in the supine position with the knee in abduction. With the patient in this position, the adductor longus muscle can be palpated before surgery. Following incision, the gracilis muscle is easily identified posterior to the adductor longus.

The adductor magnus muscle lies posterior to the gracilis, and the pectineus muscle is found anterior to the adductor longus at the floor of the femoral triangle. The sartorius muscle overlies the gracilis muscle distally.

106
Q

A 25-year-old woman who sustained the forearm avulsion shown in the photograph above subsequently underwent reconstruction using a free groin flap based on the superficial circumflex iliac artery. Which of the following best describes the vascular anatomy of this flap?

(A) The superficial circumflex iliac artery arises directly from the external iliac artery in approximately 85% of patients
(B) The superficial circumflex iliac artery arises from a common trunk, terminally splitting with the superficial inferior epigastric artery in approximately 70% of patients
(C) The superficial circumflex iliac and superficial inferior epigastric arteries have separate origins in approximately 40% of patients
(D) The superficial circumflex iliac artery is generally found approximately 1 cm below the inguinal ligament in approximately 70% of patients

A

The correct response is Option C.

The free groin flap is typically an axially patterned flap that receives its vascularity by the superficial circumflex iliac artery, which arises from the common or superficial femoral artery and then traverses laterally, parallel to the inguinal ligament, typically 2 to 3 cm inferior to the ligament. Although it provides excellent thin soft-tissue coverage of cutaneous defects and is associated with minimal donor site morbidity, especially in women, its use is limited by potential variations in vascular anatomy, as shown in the illustration below.

In 45% to 50% of persons the superficial circumflex iliac artery and superficial inferior epigastric artery arise from a common trunk, as shown in the figure on the left. In contrast, 40% to 45% of persons have a superficial circumflex iliac artery and superficial inferior epigastric artery that arise from separate origins, as shown in the figure on the right. The middle figure demonstrates a large superficial circumflex iliac artery without a superficial inferior epigastric artery, which is present in 10% to 15% of persons. In patients being considered for reconstructive procedures using the free groin flap, vascular anatomy can be determined preoperatively using Doppler ultrasonography.

In addition to its usefulness in coverage of cutaneous defects, as shown in the postoperative photograph below, the free groin flap can also be deepithelialized and transferred as soft-tissue fill in patients with Romberg’s disease or hemifacial microsomia.

107
Q

A Z-plasty revision procedure is to be performed for lengthening of a scar contracture. In order to achieve a theoretical 100% gain in the length, the angle of the Z-plasty should be how many degrees?

(A) 30
(B) 45
(C) 60
(D) 75
(E) 90

A

The correct response is Option D.

The Z-plasty is a technique in which pairs of triangular transposition flaps are created adjacent to a scar and then transposed across the scar, resulting in an increase in the length of the central limb and a change in the orientation of the scar. This technique can be used in patients undergoing burn reconstruction to lengthen linear scar contractures, disperse linear scars, and realign the scars within the lines of minimal tension. The actual amount of scar lengthening correlates directly with the angle and length of the flap limbs, as demonstrated in the table above.

108
Q

After undergoing radical mastectomy of the left breast for management of breast carcinoma, a 40-year-old woman with obesity is scheduled for delayed reconstruction using a transverse rectus abdominis myocutaneous (TRAM) flap. Which of the following is the most likely sequela of a delayed TRAM flap procedure?

(A) Increased blood flow to the deep inferior epigastric artery
(B) Increased diameter of the superior epigastric artery
(C) Increased pressure within the superior epigastric vein
(D) Increased quantity of choke vessels
(E) Increased quantity of myocutaneous perforators

A

The correct response is Option B.

A delay procedure is appropriate for any patient considering TRAM flap reconstruction who has risk factors for flap loss, including obesity, a smoking history, a previous history of radiation therapy, or large volume requirements. A delayed procedure is typically performed in the outpatient setting and involves ligation of the deep and superficial inferior epigastric vessels, eliminating blood flow through the deep inferior epigastric artery. Studies of patients who have undergone this procedure demonstrated increased diameter and flow volume of the superior epigastric artery. The vascular structures within the flap are also dilated during the delay procedure. The choke vessels connect adjacent vascular regions and have been shown in animal studies to achieve maximum dilation 48 to 72 hours after surgery. Cell hypertrophy has been demonstrated within the walls of the choke vessels.

109
Q

A 53-year-old man has a chronic draining sinus of the perineal region one year after undergoing abdominoperineal resection of a low-lying rectal carcinoma followed by localized radiation therapy. Following debridement of the affected area, which of the following is the most appropriate management?

(A) Healing by secondary intention
(B) Primary closure
(C) Skin grafting
(D) Coverage with a fasciocutaneous flap
(E) Coverage with a muscle flap

A

The correct response is Option E.

In a patient who has a chronic, irradiated wound, the most appropriate management is coverage with a muscle or musculocutaneous flap with a vascular pedicle that lies outside the field of radiation, such as the gracilis flap. Secondary intention healing is unlikely to be successful in a radiated wound bed, and primary closure of a previously radiated, nonhealing wound will also not result in appropriate healing. Skin grafting will be ineffective due to the poor vascularity of the wound bed. A fasciocutaneous flap is less appropriate than a muscle flap to fill the dead space within the wound.

110
Q

A 53-year-old man has a chronic draining sinus of the perineal region one year after undergoing abdominoperineal resection of a low-lying rectal carcinoma followed by localized radiation therapy. Following debridement of the affected area, which of the following is the most appropriate management?

(A) Healing by secondary intention
(B) Primary closure
(C) Skin grafting
(D) Coverage with a fasciocutaneous flap
(E) Coverage with a muscle flap

A

The correct response is Option E.

In a patient who has a chronic, irradiated wound, the most appropriate management is coverage with a muscle or musculocutaneous flap with a vascular pedicle that lies outside the field of radiation, such as the gracilis flap. Secondary intention healing is unlikely to be successful in a radiated wound bed, and primary closure of a previously radiated, nonhealing wound will also not result in appropriate healing. Skin grafting will be ineffective due to the poor vascularity of the wound bed. A fasciocutaneous flap is less appropriate than a muscle flap to fill the dead space within the wound.

111
Q

Which of the following is NOT an indication for Z-plasty?

(A) Adjusting soft-tissue contour
(B) Dispersing linear scars
(C) Lengthening linear scar contractures
(D) Preventing burn scar contractures

A

The correct response is Option D.

The Z-plasty is a fundamental plastic surgery technique that involves the elevation and interposition of two equal, interposed triangular skin flaps. This procedure can be used to improve soft-tissue contour via reorientation of skin, realign scars within the lines of minimal tension, and lengthen linear scar contractures. However, it is not performed for prevention of burn scar contractures.

112
Q

A 60-year-old man has a patent but widely exposed Gore-Tex dialysis access graft in the antecubital fossa. A photograph is shown above. Appropriate coverage of this defect is best accomplished using a flap that is vascularized by which of the following structures?

(A) Deep inferior epigastric artery
(B) Radial recurrent artery
(C) Septal branches of the profunda brachii artery
(D) Superficial circumflex iliac artery
(E) Ulnar artery

A

B?

113
Q

Which of the following structures provides the vascular supply to the osteocutaneous radial forearm flap?

(A) Fascioperiosteal perforators between the flexor carpi radialis and palmaris longus muscles
(B) Fascioperiosteal perforators between the brachioradialis and flexor carpi radialis muscles
(C) Musculoperiosteal perforators from the recurrent radial artery
(D) Musculoperiosteal perforators through the flexor digitorum profundus muscle
(E) Musculoperiosteal perforators through the pronator teres muscle

A

The correct response is Option B.

During harvest of the osteocutaneous radial forearm flap, a segment of radius as long as 10 cm with as much as 40% of the cross-section of the radius can be harvested. This flap can be found between the insertion of the pronator teres and brachioradialis muscles and is raised from the radial and ulnar sides. Vascularity is primarily supplied by fascioperiosteal perforators that lie within the intermuscular septum between the brachioradialis and flexor carpi radialis muscles. Musculoperiosteal branches of the flexor pollicis longus and perforators in the pronator quadratus muscle arising from the radial artery also supply blood to the flap. Dissection is performed laterally to the anterolateral intermuscular septum between the flexor carpi radialis and brachioradialis muscles and medially to the medial side of the intermuscular septum between the brachioradialis and flexor carpi radialis muscles. The brachioradialis is retracted radially and the flexor carpi radialis is retracted medially. The bellies of the flexor pollicis longus and pronator quadratus muscles are divided longitudinally to the radial periosteum, and the periosteum is incised beyond the attachments of the septum to the radius. Bone instruments are used to remove a segment of radius attached to the intermuscular septum and to the remnants of the pronator quadratus and flexor pollicis longus.

The recurrent radial artery is part of the anastomotic vasculature that surrounds the elbow. The fascioperiosteal perforators lie within the intermuscular septum between the brachioradialis and flexor carpi radialis muscles, not the flexor carpi radialis and palmaris longus muscles. There are no functional perforators arising from the flexor digitorum profundus or pronator teres muscles to supply blood to the radius.

114
Q

A 56-year-old man has a deep soft-tissue defect of the posterior neck with exposure of the vertebral bone after undergoing excision of a malignant tumor. Which of the following would preclude the use of a trapezius flap for coverage of the defect?

(A) Atherosclerotic occlusion of the occipital arteries
(B) Atherosclerotic occlusion of the vertebral arteries
(C) Prior ipsilateral carotid endarterectomy
(D) Prior ipsilateral radical neck dissection
(E) Prior ligation of the ipsilateral circumflex scapular vessels

A

The correct response is Option D.

The transverse cervical artery, which provides the primary vascular supply to the trapezius flap, is typically divided during an ipsilateral radical neck dissection. Therefore, the trapezius flap cannot be used for coverage of a defect in a patient who has undergone an ipsilateral radical neck dissection because its primarily vascular supply is presumed hes. The anterior branch then courses toward the shoulder, while the posterior branch courses beneath the central portion of the trapezius along its main axis, continuing to supply blood to this flat, triangularly shaped type II muscle.

The occipital artery is a secondary source of vascularity for the trapezius muscle flap. However, in looking at the specific location and depth of this patient’s defect, it appears that the portion of the muscle supplied by the occipital artery has been resected. The medial edge of the trapezius flap also receives some blood from posterior thoracic intercostal perforators along the medial edge of the flap.

Because carotid endarterectomy is performed at or close to the bifurcation of the common carotid artery, it does not disrupt the thyrocervical trunk. The circumflex scapular vessels and vertebral arteries are not involved in supplying vascularity to the trapezius flap.

115
Q

In a patient who has undergone resection of a squamous cell carcinoma of the floor of the mouth, which of the following free flaps will provide vascularized bone and a sensate skin paddle?

(A) Iliac crest flap
(B) Lateral arm flap
(C) Parascapular flap
(D) Serratus anterior flap

A

The correct response is Option B.

Because the lateral arm flap provides both vascularized bone and a sensate skin paddle, it is best used for reconstruction of this patient’s defect involving the floor of the mouth. Vascularity and skin sensibility are provided by the radial collateral artery and posterior brachial cutaneous nerve (C5-6), respectively. As much as 7 cm * 12 cm of skin can be elevated with the flap; in addition, because of its periosteal attachments, as much as one-third of the posterior lateral humerus (or 10 cm to 15 cm in length and 1 cm to 1.5 cm in diameter) can be harvested.

The iliac crest osteocutaneous flap, which is based on the deep circumflex iliac artery, can provide a skin paddle as large as 12 cm * 6 cm and a bone segment as large as 8 cm ( 18 cm. Although this flap can be used for reconstruction of large mandibular segments and extensive soft-tissue defects, the skin component is bulky and insensate. Meticulous closure of the donor site defect is required to prevent hernia formation.

The parascapular flap is based on the circumflex scapular artery. Advantages of this flap include multiple skin paddles, a large segment of bone, and a high degree of independent motion between the skin and bone segments. The serratus anterior and/or latissimus dorsi muscles can be included with the flap to reconstruct complex defects. However, the skin paddles of this flap are also bulky and lack a cutaneous sensory nerve.

The serratus anterior flap is extremely versatile. Skin, muscle, and an iliac bone graft can be included with this flap; its pedicle is long and has a large diameter. It can be harvested as a functional muscle flap with inclusion of the branches of the long thoracic nerve; however, the upper four to five muscle slips must be preserved in order to prevent winging of the scapula. This primary disadvantage of this flap is that any bone incorporated with it will be less substantial and have poor vascularization when compared with other osteocutaneous flaps. The skin component of this flap is also insensate.

116
Q

A 21-year-old man sustains an avulsion injury involving the skin of the dorsal aspect of the right hand. On examination, there is a loss of paratenon; the extensor tendons are exposed. A reverse radial forearm flap is to be used for coverage of the defect.

The venous outflow of this flap depends primarily on which of the following vessels?

(A) Accessory cephalic vein
(B) Basilic vein
(C) Cephalic vein
(D) Radial venae comitantes
(E) Ulnar venae comitantes

A

The correct response is Option D.

All reverse flaps are based on a “retrograde flow” design, in which the blood flows into the artery and out of the vein, opposing normal physiology. In order for this to occur properly, either the valves of the peripheral veins must be incompetent or the flow of blood must circumvent the valves. The reverse radial forearm flap has small, intercommunicating veins that lie between the paired venae comitantes and act as shunts, bypassing the valves and allowing blood flow directly between the venae comitantes. Thus, retrograde flow is established, and a useful, viable vascularized flap is created.

As long as the paired venae comitantes and intercommunicating veins remain intact, the reverse radial forearm flap can be used to cover large defects involving the dorsal aspect of the hand. Any mechanical separation of the venae comitantes during flap harvest will result in damage to the intercommunicating veins and lead to flap failure. In addition, an Allen’s test should be performed preoperatively to demonstrate an adequate retrograde flow of blood from the ulnar artery through the distal artery.

The cephalic vein originates at the radial aspect of the dorsal venous network and flows into the axillary vein. The basilic vein originates ulnarly within the dorsal venous network, joins with the median cubital vein, and ascends medial to the biceps tendon, perforating the deep fascia at the middle of the arm and then continuing as the axillary vein. The median vein of the forearm provides drainage for the superficial palmar venous plexus and ascends on the volar aspect of the forearm to drain into the basilic or median cubital vein. Because these veins and the accessory cephalic vein do not have interconnecting veins that allow for bypass of the valves, reverse flow is prohibited.

The ulnar venae comitantes act in conjunction with the ulnar artery and thus are not involved in the drainage of the reverse radial forearm flap.

117
Q

A 57-year-old man undergoes composite resection of an advanced squamous cell carcinoma of the retromolar trigone. An osteocutaneous free flap that provides a 6-cm bone segment, intraoral lining, and external skin will be used for reconstruction of the defect.

Which of the following osteocutaneous free flaps will allow for maximum independence in repositioning the skin paddle in relation to the bone segment?

(A) Fibular
(B) Iliac crest
(C) Lateral arm
(D) Radial forearm
(E) Scapular

A

The correct response is Option E.

Reconstruction in this patient should be performed using the scapular flap, which will provide the greatest degree of leeway in positioning the skin paddle in relation to the bone segment. This is typically advantageous when using the flap to reconstruct complex defects of the head and neck. The scapular flap derives its primary blood supply from the circumflex scapular artery, which originates from the subscapular artery and passes through the triangular space. Branches of the circumflex scapular artery consistently supply the lateral border of the scapula at a point prior to the division of the artery into transverse and descending branches. A 3-cm vascular pedicle extends from the border of the scapula to the overlying skin and allows for an additional three degrees of spatial freedom when insetting the skin paddle. In addition, the angular branch of the thoracodorsal artery has been shown to consistently provide an independent source of perfusion to the inferior pole of the scapula. This allows for a greater arc of rotation between the bone and skin paddle because each portion derives its vascularity from separate sources.

The fibular flap is based on the peroneal artery as well as its multiple periosteal vessels and cutaneous perforators; it can provide as much as 25 cm of bone for mandibular reconstruction. Because of the limited amount of skin that can be harvested with this flap, it is not often used in reconstruction without additional skin grafting; in addition, there is very little freedom in repositioning the skin relative to the bone. Harvest of this flap is contraindicated in patients with several peripheral vascular disease.

The iliac crest osteocutaneous flap, which is based on the deep circumflex iliac artery, can provide a skin paddle as large as 12 cm * 6 cm and a bone segment as large as 8 cm * 18 cm. Although this flap can be used for reconstruction of large mandibular segments and extensive soft-tissue defects, the skin component is bulky and insensate. Meticulous closure of the donor site defect is required to prevent hernia formation.

Advantages of the lateral arm flap include a thin, pliable, sensate skin paddle and the potential harvest of as much as 12 cm of skin and one third of the diameter of the humerus. Because of these factors, the flap is often used for reconstruction of defects of the head and neck or upper extremities. A skin paddle as large as 6 cm can be harvested without skin grafting. However, the use of this flap is once again limited by its lack of freedom between the skin and underlying bone segment.

The radial forearm flap can provide thin, pliable skin and a maximum of 10 cm of bone, which can include a cross-sectional area comprising approximately 40% of the radius. Once again, there is minimal freedom when positioning

118
Q

A 25-year-old woman sustains a contact injury to the posterior aspect of the scalp. Following debridement, she has a 6 * 4-cm defect of the posterior scalp with exposed bone. Which of the following is the most appropriate next step in management?

(A) Excision and primary closure
(B) Full-thickness skin grafting
(C) Coverage with a rotation flap
(D) Hair transplantation
(E) Tissue expansion

A

The correct response is Option C.

In this 25-year-old woman who has a 6 * 4-cm defect of the posterior aspect of the scalp, the most appropriate management is coverage of the defect using a rotation flap. This flap provides local hair-bearing tissue and can be used to cover defects as large as 6 cm. In order to advance an adequate length of flap, multiple relaxing incisions must be performed within the galea. If the galea is not carefully divided, injury to the subcutaneous vessels or hair follicles may result, leading to the onset of alopecia or delayed wound healing.

Tissue expansion is most appropriate for patients who have large defects of the scalp (typically greater than 15%) because the scalp defect will be covered with similar tissue. Donor site scarring is not a factor in most cases, and the expanders can be left in place if further tissue expansion is required. In addition, the hair follicles will be oriented correctly; as much as 50% of the scalp can be covered with expanded tissue without altering hair growth. However, the process of tissue expansion involves multiple procedures and frequent office visits over a lengthy period of time. During the expansion process, the patient often expresses displeasure with his/her physical appearance.

Excision and primary closure combined with extensive undermining are only appropriate for patients who have defects measuring less than 5 cm. Patients with small areas of scalp alopecia may undergo multiple staged excisions of the alopecic scalp followed by advancement of hair-bearing tissue. Because this process involves fewer procedures and less follow-up, it can be used as an alternative to tissue expansion.

Although a full-thickness skin graft can be used for temporary wound coverage in a patient who will undergo further scalp reconstruction, it is not an appropriate long-term treatment because many donor sites do not provide adequate hair-bearing skin.

Hair transplantation is currently being used with increasing regularity for treatment of traumatic or age-related alopecia.

119
Q

Which of the following structures provides motor innervation to the gracilis free muscle flap?

(A) Anterior branch of the obturator nerve
(B) Femoral nerve
(C) Inferior branch of the superior gluteal nerve
(D) Medial femoral cutaneous nerve
(E) Median sural nerve

A

The correct response is Option A.

The anterior branch of the obturator nerve provides motor innervation to the gracilis free muscle flap. This nerve branch courses between the adductor longus and adductor brevis tendons to innervate the gracilis muscle.

The femoral nerve innervates the rectus femoris muscle at the level of the thigh, while the inferior branch of the superior gluteal nerve supplies motor innervation to the tensor fascia lata. The medial femoral cutaneous nerve, which is a branch of the femoral nerve, supplies sensory innervation to the medial thigh flap. The median sural nerve is found below the knee and courses parallel to the lesser saphenous vein.

120
Q

A vastus lateralis muscle flap elevated on its dominant pedicle provides reliable coverage for each of the following anatomic sites EXCEPT the

(A) acetabulum
(B) groin
(C) knee
(D) perineum
(E) trochanter

A

The correct response is Option C.

When the vastus lateralis flap is based on its dominant pedicle, the descending branch of the lateral femoral circumflex artery, it has an area of rotation that will provide vascularized coverage of the lower abdomen, groin, perineum, ischium, trochanter, and acetabular fossa. However, the flap must be reversed in order to rotate and provide coverage of knee defects. When used in this manner, the flap is then based on a branch of the lateral genicular artery, which is a minor distal pedicle. Because the risk for partial flap loss is greater, this flap is not often advocated for coverage of knee defects.

121
Q

Each of the following is an effective technique for continuous postoperative free flap monitoring EXCEPT

(A) differential surface temperature monitoring
(B) external Doppler ultrasonography
(C) intravenous injection of fluorescein
(D) laser Doppler ultrasonography
(E) photoplethysmography

A

The correct response is Option C.

Although fluorescein 15 mg/kg is often administered intravenously to determine the viability of a flap’s skin paddles, this cannot be used for continuous free flap monitoring because fluorescein often takes several hours to clear from the skin. Lower doses of fluorescein can be used for sequential monitoring, but not for continuous monitoring.

Each of the other options listed can be used for continuous flap monitoring. Although external Doppler ultrasonography is the most frequent choice for flap monitoring, difficulties can be experienced when the flap pedicle is located close to a large artery. External Doppler ultrasonography also cannot be used with buried flaps. Differential surface temperature monitoring compares the temperature of the transferred tissue with the normal surrounding tissue. A temperature difference of greater than 1.8%C (35.3%F) is believed to be significant. Laser Doppler ultrasonography and photoplethysmography both involve the measurement of reflected light from a source that penetrates the flap.

122
Q

A 25-year-old man sustains an extravasation injury of the dorsal aspect of the wrist. Following debridement, the extensor tendons are exposed; a photograph is shown above. Findings on Allen’s test demonstrate radial dominance. Which of the following is the most appropriate next step in management?

(A) Dressing changes and healing by second intention
(B) Split-thickness skin grafting
(C) Coverage with a free lateral arm flap
(D) Coverage with a free rectus abdominis muscle flap and split-thickness skin graft
(E) Coverage with a reverse radial forearm flap

A

The correct response is Option C.

In this patient who has exposure of the tendons and loss of paratenon after sustaining a wrist injury, the most appropriate management is coverage of the defect using a free lateral arm flap. This is a pliable fasciocutaneous flap that will provide durable coverage of this patient’s defect with optimal cosmetic results and minimal donor site morbidity. Its good gliding surface will permit tendon gliding without tethering. If tethering were to occur subsequently, a secondary tenolysis could be easily performed underneath this thin flap.

Dressing changes and healing by second intention are not appropriate in a patient with exposed tendons because tendon adhesions and/or dessication will result. In the same way, the defect should not be covered with a split-thickness skin graft, which will not take when placed directly on top of exposed tendon. Coverage with a free rectus abdominis flap will be bulky and can result in tethering. Secondary tenolysis will be difficult under this flap. The reverse radial forearm flap has been linked to the development of hand ischemia in patients who demonstrate radial dominance on Allen’s testing.